SlideShare ist ein Scribd-Unternehmen logo
1 von 22
Downloaden Sie, um offline zu lesen
Gripping IFRS                          Non-current assets held for sale and discontinued operations



                                         Chapter 9
            Non-current Assets Held for Sale and Discontinued Operations

Reference: IFRS 5

Contents:                                                                                    Page

   1. Introduction                                                                            310
   2. Definitions                                                                             310
   3. Non-current assets held for sale: identification                                        311
      3.1 Overview                                                                            311
      3.2 Criteria to be met before a non-current asset is classified as ‘held for sale’      311
          3.2.1 General criteria                                                              311
          3.2.2 Criteria where a completed sale is not expected within one year               311
          3.2.3 Criteria where the asset is acquired with the intention to sell               312
   4. Non-current assets held for sale: measurement                                           312
      4.1 General measurement principles                                                      312
      4.2 Measurement principles specific to the cost model                                   313
          4.2.1 The basic principles when the cost model was used                             313
          Example 1: reclassification of an asset measured using the cost model               313
          Example 2: reclassification of an asset measured using the cost model               315
          4.2.2 The tax effect when the cost model was used                                   316
          Example 3: tax effect of reclassification and the cost model                        316
      4.3 Measurement principles specific to the revaluation model                            317
          4.3.1 The principles when the revaluation model was used                            317
          Example 4: reclassification of an asset using the revaluation model                 318
          Example 5: re-measurement of an asset held for sale, using the                      320
          revaluation model
      4.4 Reversal of classification as ‘held for sale’                                       321
          Example 6: re-measurement of assets no longer ‘held for sale’                       322

   5. Non-current assets held for sale: disclosure                                            322
      5.1 Overview                                                                            322
      5.2 In the statement of financial position                                              322
      5.3 In the statement of financial position or notes thereto                             323
      5.4 Other note disclosure                                                               323
          Example 7: disclosure of non-current assets held for sale                           323

   6. Discontinued operations: identification                                                 325
   7. Discontinued operations: measurement                                                    325
   8. Discontinued operations: disclosure                                                     326
      8.1 In the statement of comprehensive income                                            326
      8.2 In the statement of cash flows                                                      327
      8.3 Other note disclosure                                                               328
          8.3.1 Components no longer held for sale                                            328
          8.3.2 Criteria met after the end of the reporting period                            328
   9. Summary                                                                                 329




                                             309                                        Chapter 9
Gripping IFRS                           Non-current assets held for sale and discontinued operations



1. Introduction

As its name suggests, this IFRS covers two areas, namely:
• non-current assets held for sale; and
• discontinued operations.

With regard to ‘non-current assets’, this IFRS essentially suggests that there needs to be a
further classification in the statement of financial position: ‘non-current assets held for sale’.
In addition, it specifies that ‘held for sale assets’ are not to be depreciated.

This IFRS does not apply to the following assets since these assets are covered by their own
specific standards:
• Deferred tax assets (IAS 12)
• Assets relating to employee benefits (IAS 19)
• Financial assets (IAS 39)
• Investment property measured under the fair value model (IAS 40)
• Non-current assets measured at fair value less point-of-sale costs (IAS 41: Agriculture)
• Contractual rights under insurance contracts (IFRS 4)

2. Definitions

Definitions included in Appendix A of the IFRS include the following:
•   Current asset: an asset
     - that is expected to be realised within 12 months after the end of the reporting period;
     - that is expected to be sold, used or realised (converted into cash) as part of the
        normal operating cycle;
     - that is held mainly for the purpose of being traded; or
     - that is a cash or cash equivalents that is not restricted in use within the 12 month
        period after the end of the reporting period.
•   non-current asset: an asset that does not meet the definition of a current asset
•   discontinued operation: a component of an entity that either has been disposed of or is
    classified as held for sale and:
    a) represents a separate major line of business or geographical area of operations,
    b) is part of a single co-ordinated plan to dispose of a separate major line of business or
        geographical area of operations; or
    c) is a subsidiary acquired exclusively with view to resale.
•   component of an entity: operations and cash flows that can be clearly distinguished,
    operationally and for financial reporting purposes, from the rest of the entity.
•   disposal group: a group of assets to be disposed of, by sale or otherwise, together as a
    group in a single transaction, and liabilities directly associated with those assets that will
    be transferred in the transaction. The group includes goodwill acquired in a business
    combination if the group is a cash-generating unit to which goodwill has been allocated in
    accordance with the requirements of paragraphs 80-87 of IAS 36 Impairment of Assets
    (as revised in 2004) or if it is an operation within a cash-generating unit.
•   firm purchase commitment: an agreement with an unrelated party, binding on both
    parties and usually legally enforceable, that:
    a) specifies all significant terms, including the price and timing of the transactions; and
    b) includes a disincentive for non-performance that is sufficiently large to make
        performance highly probable.
•   highly probable: significantly more likely than probable.
•   probable: more likely than not.

                                               310                                       Chapter 9
Gripping IFRS                             Non-current assets held for sale and discontinued operations




3.      Non-current assets held for sale: identification (IFRS 5.6 - .12)

3.1     Overview

The main thrust of IFRS 5 is that non-current assets that are ‘held for sale’ must be classified
separately in the statement of financial position (i.e. a machine that is held for sale will no
longer be included as part of property, plant and equipment). Certain criteria must first be
met before a non-current asset is classified as a ‘non-current asset held for sale’.

3.2     Criteria to be met before a non-current asset is classified as ‘held for sale’

3.2.1 General criteria

A non-current asset (or disposal group) must be classified as held for sale if its carrying
amount will be recovered mainly through a sale transaction than through continuing use.

Non-current assets that meet all the following criteria may be separately classified as ‘non-
current assets held for sale’:
• Is the asset available for sale immediately and at normal terms? The asset (or disposal
    group) must be available for immediate sale in its present condition subject only to terms
    that are usual and customary for sales of such assets (or disposal groups);
• Has management committed itself to a sales plan? Management, with the necessary
    authority to approve the action, must have committed itself to a plan to sell;
• Has an active programme to sell begun? The active programme must be to both locate a
    buyer and to complete the plan to sell the asset (or disposal group);
• Is the sale expected to happen within one year? The sale must be expected to qualify for
    recognition as a completed sale within one year from the date of classification as held for
    sale, except as permitted by paragraph 9 and appendix B;
• Is the expected selling price reasonable? The asset (or disposal group) must be actively
    marketed at a price that is reasonable in relation to its current fair value; and
• Is it unlikely that significant changes to the plan will be made? The actions required to
    complete the plan must indicate that it is unlikely that significant changes to the plan will
    be made or that the plan will be withdrawn.

This means that assets that are to be abandoned should not be classified and measured as
‘held for sale’ since their carrying amount will be recovered principally through continuing
use (until date of abandonment) rather than through a sale. This means that depreciation on
assets that are to be abandoned should not cease.

3.2.2 Criteria where a completed sale is not expected within one year (Appendix B)

There may be occasions where the asset would still be ‘held for sale’ even though the sale
may not be completed and recognised as a sale within one year. This happens when:
•     At the date that the entity commits itself to a plan to sell a non-current asset (or disposal
      group), it reasonably expects that others (not a buyer) will impose conditions on the
      transfer of the asset (or disposal group) that will extend the period required to complete
      the sale, and:
      - actions necessary to respond to those conditions cannot be initiated until after a firm
          purchase commitment is obtained, and
      - a firm purchase commitment is highly probable within one year.
•     An entity obtains a firm purchase commitment and, as a result, a buyer or others
      unexpectedly impose conditions on the transfer of a non-current asset (or disposal group)
      previously classified as held for sale that will extend the period required to complete the
      sale, and:
      - timely actions necessary to respond to the conditions have been taken, and
      - a favourable resolution of the delaying factors is expected.

                                                311                                        Chapter 9
Gripping IFRS                            Non-current assets held for sale and discontinued operations



•     During the initial one-year period, circumstances arise that were previously considered
      unlikely and, as a result, a non-current asset (or disposal group) previously classified as
      held for sale is not sold by the end of the period, and:
      - during the initial one-year period the entity took action necessary to respond to the
          change in circumstances,
      - the non-current asset (or disposal group) is being actively marketed at a price that is
          reasonable, given the change in circumstances, and
      - the criteria in paragraph 7 (that sets out that the asset must be available for immediate
          sale) and paragraph 8 (that sets out that the sale must be highly probable) are met.

3.2.3 Criteria where the asset is acquired with the intention to sell (IFRS 5.11)

It may happen that an entity acquires a non-current asset (or disposal group) exclusively with
the view to its subsequent disposal. In this case, the non-current asset must be classified as
‘held for sale’ immediately on acquisition date, on condition that:
• the one-year requirement is met (unless a longer period is allowed by paragraph 9 and the
    related appendix B); and
• it is highly probable that any other criteria given in para 7 and para 8 that are not met
    immediately on the date of acquisition, will be met within a short period (usually three
    months) after acquisition.

4.      Non-current assets held for sale: measurement (IFRS 5.15 - .25)

4.1     General measurement principles

An entity shall measure a non-current asset (or disposal group) classified as held for sale at
the lower of its carrying amount and fair value less costs to sell.

If a newly acquired asset (or disposal group) meets the criteria to be classified as held for sale,
applying paragraph 15 will result in the asset being measured on initial recognition at the
lower of its carrying amount had it not been so classified (e.g. cost) and fair value less costs to
sell. Since the asset is newly acquired, its cost will equal its fair value. Therefore, an asset
acquired as part of a business combination, shall initially be measured at fair value (its cost)
less costs to sell.

For all other assets (other than newly acquired assets) that are classified as non-current assets
held for sale, there are two distinct phases of its life:
 • Before it was classified as held for sale; and
 • Once it is classified as held for sale.

Before an asset is classified as held for sale, it is measured in terms of its own relevant IFRS.
If, for example, the asset is an item of property, plant and equipment, the asset will have been
measured in terms of IAS 16, which will mean that:
• on initial acquisition, the asset will have been recorded at cost; and
• subsequently, the asset will have been depreciated, revalued (if the revaluation model
     was used to measure the asset) and reviewed for impairments annually (whether the cost
     or revaluation model were used).

If this asset is then to be reclassified as ‘held for sale’, it will be measured as follows:
• In terms of its previous relevant IFRS:
     Immediately before reclassifying the asset as ‘held for sale’, the asset must be re-
     measured using its previous measurement model; for example if the asset was previously
     an item of property, plant and equipment that was measured using the:
    • Cost model: depreciate to date of reclassification and then check for impairments; or
    • Revaluation model: depreciate to date of reclassification, revalue if appropriate and
         check for impairments; then


                                               312                                        Chapter 9
Gripping IFRS                             Non-current assets held for sale and discontinued operations



•     In terms of IFRS 5:
      On reclassifying the asset as ‘held for sale’,
       - re-measure to the lower of ‘carrying amount’ and ‘fair value less costs to sell’; and
       - stop depreciating it.

If, in the unusual instance a sale is not expected to occur within one year, it may be necessary
(depending on materiality) to measure the ‘costs to sell’ at their present value.

4.2    Measurement principles specific to the cost model

4.2.1 The basic principles when the cost model was used

If an asset measured under the cost model is re-classified as ‘held for sale’:
• immediately before reclassifying the asset as ‘held for sale’, the asset must be re-
     measured using its previous measurement model (i.e. the cost model per IAS 16, if the
     item was previously property, plant and equipment);
• then, in terms of IFRS 5:
     - re-measure it to the lower of ‘carrying amount’ and ‘fair value less costs to sell’;
     - stop depreciating it; and
     - re-measure to ‘fair value less costs to sell’ whenever appropriate: any impairment loss
         will be expensed in the statement of comprehensive income whereas impairment
         losses reversed are recognised as income but are limited to the asset’s accumulated
         impairment losses.

You may have noticed that, when using the cost model, there can be no initial increase in the
carrying amount on classification as ‘held for sale’ because the non-current asset must
initially be measured at the lower of its ‘carrying amount’ and ‘fair value less costs to sell’.
For example, an asset with a ‘carrying amount’ of 80 000 and ‘fair value less costs to sell’ of
90 000 will not be adjusted because the lower of the two is the current carrying amount of
80 000.

Example 1: reclassification of an asset measured using the cost model

An item of plant, measured using the cost model, has a carrying amount of C80 000 (cost:
100 000 and accumulated depreciation: 20 000) on 1 January 20X3 on which date all criteria
for separate classification as a ‘non-current asset held for sale’ are met.

Required:
Show the journal entries relating to the reclassification of the plant assuming that:
A. the fair value is C70 000 and the expected costs to sell are C5 000 on 1 January 20X3;
B. on 30 June 20X3 (6-months later), the fair value is C70 000 and expected costs to sell are
   C2 000;
C. on 30 June 20X3 (6-months later), the fair value is C90 000 and expected costs to sell are
   C5 000.

Solution to example 1: reclassification of an asset using the cost model

Comment: this example explains the limit to the reversal of the impairment loss.

A. If carrying amount > ‘fair value less costs to sell’: recognise an ‘impairment loss’ (expense)

Workings:                                                                                     C

Carrying amount                               given                                         80 000
Fair value less costs to sell:                70 000 – 5 000                               (65 000)
Decrease in value (impairment loss)           80 000 – 65 000                               15 000



                                                 313                                       Chapter 9
Gripping IFRS                                Non-current assets held for sale and discontinued operations



Journal: 1 January 20X3                                                                Debit       Credit

Impairment loss (expense)                                                            15 000
 - Plant: accumulated impairment loss                                                              15 000
Impairment loss before initial classification as ‘held for sale’

Note: There is no depreciation on this asset.


B. If ‘fair value less costs to sell’ subsequently increases: recognise a ‘reversal of impairment loss’
   (income) – limited to accumulated impairment losses

Workings:                                                                                            C

New fair value less costs to sell:     70 000 – 2 000                                             68 000
Prior fair value less costs to sell:   100 000 cost – 20 000 accum depreciation –                (65 000)
                                       15 000 impairment loss
Impairment loss reversed*:             68 000 – 65 000                                              3 000

* Note: the ‘accumulated impairment loss’ is 15 000 before the reversal, thus the reversal of 3 000 is
not limited (the previous accumulated impairment loss is bigger: 15 000 is bigger than 3 000).

Journal: 30 June 20X3                                                                 Debit        Credit

Plant: accumulated impairment loss                                                     3 000
- Impairment loss reversed (income)                                                                 3 000
Reversal of impairment loss: on re-measurement of ‘NCA held for sale’

Note: There is no depreciation on this asset. The impairment to date is C12 000 (15 000 – 3 000)


C. If ‘fair value less costs to sell’ subsequently increases: recognise a ‘reversal of impairment loss’
   (income) – limited to accumulated impairment losses

Workings:                                                                                             C

New fair value less costs to sell:       90 000 – 5 000                                             85 000
Prior fair value less costs to sell      100 000 – 20 000 accum depreciation –                     (65 000)
                                         15 000 impairment loss
Increase in value                                                                                   20 000
Limited to prior cumulative impairment losses                                                       15 000
Impairment loss reversed*:            85 000 – 65 000 = 20 000 limited to 15 000                    15 000

* Note: the difference between the latest ‘fair value less costs to sell’ (85 000) and the prior ‘fair value
less costs to sell’ (65 000) of 20 000 is limited to the previous ‘accumulated impairment loss’ of 15
000.

Journal: 30 June 20X3                                                                 Debit        Credit

Plant: accumulated impairment loss                                                      15 000
 - Impairment loss reversed (income)                                                                 15 000
Reversal of impairment loss on re-measurement of ‘non-current asset held
for sale’

Note: There is no depreciation on this asset. The impairment to date is C0 (15 000 – 15 000)



                                                    314                                          Chapter 9
Gripping IFRS                               Non-current assets held for sale and discontinued operations



Example 2: reclassification of an asset measured using the cost model

An item of plant, measured using the cost model (i.e. at historical carrying amount), has a
carrying amount of 80 000 (cost 100 000) on 1 January 20X3 on which date all criteria for
separate classification as a ‘non-current asset held for sale’ are met. This asset had previously
been impaired by 3 000 (i.e. this is the balance on the accumulated impairment loss account).

Required:
Show the journal entries relating to the reclassification of the plant assuming:
A. the fair value is 70 000 and the expected costs to sell are 5 000 on 1 January 20X3;
B. 6 months later, on 30 June 20X3, the fair value is 70 000 and the expected costs to sell are
   2 000;
C. 6 months later, on 30 June 20X3, the fair value is 90 000 and the expected costs to sell are
   5 000.

Solution to example 2: reclassification of an asset measured using the cost model

Comment: this example explains the limit to the reversal of the impairment loss. It differs from the
previous example in that this asset had previously been impaired before it was reclassified as a non-
current asset held for sale.

A. If carrying amount > ‘fair value less costs to sell’: recognise an ‘impairment loss’ (expense)

Workings:                                                                                         C
Carrying amount                                    given                                        80 000
Fair value less costs to sell:                     70 000 – 5 000                              (65 000)
Decrease in value (impairment loss)                80 000 – 65 000                              15 000

Journal: 1 January 20X3                                                            Debit       Credit
Impairment loss (expense)                                                           15 000
 - Plant: accumulated impairment loss                                                            15 000
Impairment loss on initial classification of NCA as ‘held for sale’

Note: There is no depreciation on this asset. The impairment to date is now C18 000 (3 000 + 15 000)

B. If ‘fair value less costs to sell’ subsequently increases: recognise a ‘reversal of impairment loss’
   (income) – limited to accumulated impairment losses

Workings:                                                                                         C

New fair value less costs to sell                  70 000 – 2 000                               68 000
Prior fair value less costs to sell                70 000 – 5 000                              (65 000)
Increase in value (impairment loss reversed*)      68 000 – 65 000                               3 000

* Note: the ‘accumulated impairment loss’ is 18 000 before this reversal (15 000 + 3 000), therefore
the impairment loss reversal of 3 000 is not limited (the previous accumulated impairment loss is
bigger: 18 000 is bigger than 3 000).

Journal: 30 June 20X3                                                             Debit        Credit

Plant: accumulated impairment loss                                                 3 000
- Impairment loss reversed (income)                                                             3 000
Reversal of impairment loss on re-measurement of ‘asset held for sale’

Note: There is no depreciation on this asset. The impairment to date is now C15 000 (18 000 - 3 000)



                                                  315                                        Chapter 9
Gripping IFRS                              Non-current assets held for sale and discontinued operations



C. If ‘fair value less costs to sell’ subsequently increases: recognise a ‘reversal of impairment loss’
   (income) – limited to accumulated impairment losses

Workings:                                                                                          C

New fair value less costs to sell:                90 000 – 5 000                                 85 000
Prior fair value less costs to sell               70 000 – 5 000                                (65 000)
Increase in value                                                                                20 000
Limited to prior cumulative impairment losses     15 000 + 3 000                                 18 000
Impairment loss reversed*:                        85 000 – 65 000 = 20 000 limited to 15 000     18 000

* Note: The difference between the latest ‘fair value less costs to sell’ and the prior ‘fair value less
costs to sell’ of 20 000 is limited to the ‘cumulative impairment loss’ recognised of 18 000, calculated
as follows:
                                                                                                  C
         Impairment loss:                                                                       18 000
          - before reclassification                  given                                       3 000
          - on reclassification                      80 000 – 65 000                            15 000


Journal: 30 June 20X3                                                              Debit        Credit

Plant: accumulated impairment loss                                                 18 000
- Impairment loss reversed (income)                                                              18 000
Reversal of impairment loss on re-measurement of ‘asset held for sale’

Note: There is no depreciation on this asset. The impairment to date is now C0 (18 000 - 18 000)

4.2.2 The tax effect when the cost model was used

As soon as an asset is classified as held for sale, depreciation thereon ceases. The tax
authorities, however, do not stop deducting tax allowances (where tax allowances were due in
terms of the tax legislation) simply because you have decided to sell the asset. The difference
between the nil depreciation and the tax allowance (if appropriate) causes deferred tax. The
principles affecting the current tax payable and deferred tax balances are therefore exactly the
same as for any other non-current asset.

Example 3: tax effect of reclassification and the cost model

An item of plant, measured using the cost model (i.e. at historical carrying amount), has a
carrying amount of C70 000 (cost 100 000) and a tax base of C90 000 on 1 January 20X3 on
which date all criteria for separate classification as a ‘non-current asset held for sale’ are met.
The fair value less costs to sell on this date are C65 000. This asset had not previously been
impaired. The tax authorities allow a deduction of 10% on the cost of this asset. The tax rate
is 30%. The profit before tax is correctly calculated to be C200 000. There are no temporary
or permanent differences other than those evident from the information provided.

Required:
A. Calculate the current normal tax payable and the deferred tax balance at 31 December
   20X3.
B. Journalise the current normal tax and the deferred tax for the year ended 31 December
   20X3.




                                                 316                                           Chapter 9
Gripping IFRS                                Non-current assets held for sale and discontinued operations



Solution to example 3: tax effect of reclassification and the cost model

A: Calculations

Current normal income tax          Calculations                                                       C

Profit before tax                                                                                   200 000
Add back depreciation              Assets held for sale are not depreciated                               0
Add back impairment                Impairment on re-classification as ‘held for sale’                 5 000
Less tax allowance                 100 000 x 10%                                                    (10 000)
Taxable profits                                                                                     195 000
Current tax                        195 000 x 30%                                                     58 500

Deferred tax:                             Carrying          Tax         Temporary         Deferred
Non-current asset held for sale           amount            base        difference          tax

Balance – 1 January 20X3                    70 000         90 000          20 000           6 000      Asset
Less impairment to ‘fair value –
                                             (5 000)               0                                   Cr DT,
costs to sell’ (70 000 – 65 000)                                                           (1 500)
                                                                                                       Dr TE
Depreciation/ tax allowance                      0         (10 000)
Balance – 31 December 20X3                  65 000          80 000         15 000           4 500      Asset

B: Journals

31 December 20X3                                                                        Debit        Credit

Tax expense                                                                          58 500
  Current tax payable (liability)                                                                    58 500
Current normal tax payable (estimated)

Tax expense                                                                             1 500
  Deferred tax (liability)                                                                            1 500
Deferred tax adjustment


4.3     Measurement principles specific to the revaluation model

4.3.1 The principles when the revaluation model was used

If an asset measured under the revaluation model is reclassified as ‘held for sale’:

•     immediately before reclassifying the asset as ‘held for sale’, the asset must be re-
      measured using its previous measurement model (i.e. the revaluation model per IAS 16);

•      then, in terms of IFRS 5:
      - re-measure it to the lower of ‘carrying amount’ and ‘fair value less costs to sell’;
      - stop depreciating it; and
      - then re-measure it to ‘fair value less costs to sell’ whenever appropriate: any further
          impairment loss (e.g. the selling costs) is expensed (even if there is a revaluation
          surplus) whereas an impairment loss reversed is recognised as income but is limited
          to the asset’s accumulated impairment losses.




                                                     317                                        Chapter 9
Gripping IFRS                              Non-current assets held for sale and discontinued operations



Example 4: reclassification of an asset measured using the revaluation model

An item of plant, revalued to fair value using the revaluation model, met all criteria for
classification as ‘held for sale’ on 1 January 20X4. The following information is relevant:

Cost: 100 000 (purchased 1 January 20X1)
Depreciation: 10% per annum straight-line to nil residual values.
Fair value: 120 000 (revalued 1 January 20X3).
Revaluations are performed using the net replacement value method

Required:
Show all journal entries relating to the reclassification as ‘held for sale’ assuming that:
A. The fair value is C100 000 and the expected selling costs are C9 000 on 1 January 20X4;
B. The fair value is C150 000 and the expected selling costs are C20 000 on 1 January 20X4.
C. The fair value is C60 000 and the expected selling costs are C20 000 on 1 January 20X4.

Solution to example 4: reclassification of an asset measured using the revaluation model

A. If the actual carrying amount > historical carrying amount (i.e. there is already a revaluation
   surplus) and the fair value decreases on date of reclassification (although not entirely
   removing the revaluation surplus balance) and there are costs to sell: reverse revaluation
   surplus due to drop in fair value and recognise selling costs as an ‘impairment loss’ (expense)

Workings:                                                                                             C

Fair value (1 January 20X3)                                                                          120 000
Accumulated depreciation (31 December 20X3: since       120 000/ 8 remaining years                   (15 000)
the revaluation on 1 January 20X3)
Actual carrying amount (1 January 20X4):                120 000 – 15 000                             105 000
Fair value                                              Given                                       (100 000)
Decrease in value (all through revaluation surplus)     See below for calculation of RS balance        5 000

Actual carrying amount (1 January 20X4):                120 000 – 15 000 (above)                     105 000
Historical carrying amount (1 January 20X4)             100 000/ 10 years x 7 years                  (70 000)
Balance on the revaluation surplus (1 January 20X4):    Proof: (120 000 – 80 000) / 8 x 7 years       35 000
Decrease in value (above)                                                                             (5 000)
Balance on the revaluation surplus (1 January 20X4):    Further balance against which further         30 000
                                                        devaluation would be processed (IAS16)



Journals: 1 January 20X4                                                              Debit        Credit

Plant: accumulated depreciation and impairment losses                                  15 000
- Plant: cost                                                                                        15 000
NRVM: Accumulated depreciation set-off against cost

Revaluation surplus             FV: C100 000 – Carrying amount: C105 000                5 000
- Plant: cost                                                                                         5 000
Re-measurement to FV before reclassification

Impairment loss (selling costs) (expense)                                               9 000
 - Plant: accumulated depreciation and impairment losses                                              9 000
Re-measurement to lower of CA or FV less costs to sell on reclassification:
CA: 100 000 – FV less Costs to Sell: (100 000 – 9 000)

Note: There is no further depreciation on this asset.


                                                  318                                             Chapter 9
Gripping IFRS                               Non-current assets held for sale and discontinued operations



B. If the actual carrying amount > historical carrying amount (i.e. there is already a revaluation
   surplus) and fair value increases and there are expected costs to sell: increase revaluation
   surplus due to increase in fair value and recognise the expected selling costs as an ‘impairment
   loss’ (expense)

Workings:                                                                                              C

Fair value                                                                                          120 000
Accumulated depreciation (31 December 20X3:                 120 000/ 8 remaining years              (15 000)
since the revaluation on 1 January 20X3)
Actual carrying amount (1 January 20X4):                    120 000 – 15 000                        105 000
Fair value                                                  given                                   150 000
Increase in value (all through revaluation surplus)         Through revaluation surplus              (45 000)
                                                            because carrying amount is already
                                                            above the HCA: 100 000 / 10 x 7


Journals: 1 January 20X4                                                             Debit          Credit
Plant: accumulated depreciation and impairment losses                                15 000
 - Plant: cost                                                                                       15 000
NRVM: Accumulated depreciation set-off against cost: 120 000/ 8 years
remaining on date of revaluation
Plant: cost                                                                           45 000
- Revaluation surplus                                                                                45 000
Re-measurement to FV before reclassification:
FV: 150 000 – Carrying amount: 105 000
Impairment loss (selling costs) (expense)                                             20 000
 - Plant: accumulated depreciation and impairment losses                                             20 000
Re-measurement to lower of CA or FV less costs to sell on
reclassification:
Carrying amount: 150 000 – FV less costs to sell: (150 000 – 20 000)
Note: There is no further depreciation on this asset.


C. If the actual carrying amount > historical carrying amount (i.e. there is already a
   revaluation surplus) and fair value decreases removing the entire balance on the
   revaluation surplus and there are expected costs to sell: reverse revaluation surplus
   due to decrease in fair value and recognise the expected selling costs as an ‘impairment
   loss’ (expense)

Workings:                                                                                              C
Fair value                                                                                          120 000
Accumulated depreciation (31 December 20X3: since             120 000/ 8 years                      (15 000)
the revaluation on 1 January 20X3)
Actual carrying amount (1 January 20X4):                      120 000 – 15 000                      105 000
Fair value                                                    given                                  (60 000)
Decrease in value (all through revaluation surplus)           See below for calculation of RS bal     45 000

Actual carrying amount (1 January 20X4):                      120 000 – 15 000                      105 000
Historical carrying amount (1 January 20X4)                   100 000/ 10years x 7 years            (70 000)
Balance on the revaluation surplus (1 January 20X4):          (120 000 – 80 000) / 8 x 7 years       35 000
Decrease in value (above)                                                                            45 000
Reversal: revaluation surplus balance                         Balance in this account (above)        35 000
Impairment loss (balancing figure)                            45 000 – 35 000                        10 000


                                                      319                                           Chapter 9
Gripping IFRS                               Non-current assets held for sale and discontinued operations



Journals: 1 January 20X4                                                                  Debit        Credit

Plant: accumulated depreciation and impairment losses                                     15 000
 - Plant: cost                                                                                          15 000
NRVM: Accumulated depreciation set-off against cost: 120 000/ 8 years
remaining on date of revaluation
Revaluation surplus           (ACA: 105 000 – HCA: 70 000)                                35 000
Impairment loss               (HCA: 70 000 – FV: 60 000)                                  10 000
 - Plant: cost                                                                                          35 000
 - Plant: accumulated depreciation and impairment losses                                                10 000
Re-measurement to FV before reclassification: FV: 60 000 – CA: 105 000
Impairment loss (selling costs) (expense)                                                 20 000
 - Plant: accumulated depreciation and impairment losses                                                20 000
Re-measurement to lower of CA or FV less costs to sell on reclassification:
CA: 60 000 – FV less costs to sell (60 000 – 20 000)
Note: There is no further depreciation on this asset.

Example 5: re-measurement of an asset held for sale using the revaluation model

An item of plant, revalued to fair value using the revaluation model, met all criteria for
classification as ‘held for sale’ on 1 January 20X4. The following information is relevant:

Cost: 100 000 (purchased 1 January 20X1)
Depreciation: 10% per annum straight-line to nil residual values.
Fair value: 120 000 (revalued 1 January 20X3).
Revaluations are performed using the net replacement value method
The ‘fair value less costs to sell’ on 1 January 20X4 was as follows:
• Fair value (1 January 20X4): 100 000; and
• Expected selling costs (1 January 20X4): 9 000.

Required:
Show all journal entries relating to the re-measurement of the ‘non-current asset held for sale’
on 30 June 20X4 assuming that on the 30 June 20X4:
A. The fair value is 110 000 and the expected selling costs are 15 000;
B. The fair value is C110 000 and the expected selling costs are C3 000;
C. The fair value is 90 000 and the expected selling costs are 3 000.

Solution to example 5: re-measurement of an asset held for sale: the revaluation model

Comment: this example explains the limit on the impairment loss that may be reversed.

A.   If the new fair value less costs to sell > previous fair value less costs to sell:
     reverse the impairment loss limited to prior cumulative impairment losses

Workings:                                                                                                 C

New fair value less costs to sell (30 June 20X4)        110 000 (FV) – 15 000 (cost to sell)            95 000
Prior fair value less costs to sell (1 January 20X4)    100 000 (FV) – 9 000 (costs to sell)           (91 000)
Increase in value                                                                                        4 000
Limited to prior cumulative impairment losses           100 000 (FV before reclassification) –           9 000
                                                        91 000 (FV – costs to sell)
Therefore: impairment loss reversed                     Maximum that may be reversed is 9 000;           4 000
                                                        thus there is no limitation to the reversal
                                                        in this case




                                                   320                                                Chapter 9
Gripping IFRS                               Non-current assets held for sale and discontinued operations



Journals: 30 June 20X4                                                                  Debit          Credit
Plant: accumulated depreciation and impairment losses                                   4 000
 - Impairment loss reversed (income)                                                                     4 000
Re-measurement of non-current asset held for sale: increase in fair value
less costs to sell


B.    If the new fair value less costs to sell > previous fair value less costs to sell:
      reverse the impairment loss limited to prior cumulative impairment losses

Workings:                                                                                                 C
New fair value less costs to sell (30 June 20X4)       110 000 (FV) – 3 000 (cost to sell)             107 000
Prior fair value less costs to sell (1 January 20X4)   100 000 (FV) – 9 000 (costs to sell)            (91 000)
Increase in value                                                                                       16 000
Limited to prior cumulative impairment losses          100 000 (FV before reclassification) –            9 000
                                                       91 000 (FV – costs to sell)
Therefore: reversal of impairment loss                                                                   9 000

Journals: 30 June 20X4                                                                  Debit          Credit
Plant: accumulated impairment loss                                                      9 000
 - Reversal of impairment loss (income)                                                                  9 000
Re-measurement of non-current asset held for sale: increase in fair value
less costs to sell (limited to 9 000)


C.     If the new fair value less costs to sell < previous fair value less costs to sell:
       recognise a further impairment loss

Workings:                                                                                                 C
New fair value less costs to sell (30 June 20X4)       90 000 (FV) – 3 000 (cost to sell)                87 000
Prior fair value less costs to sell (1 January 20X4)   100 000 (FV) – 9 000 (costs to sell)              91 000
Decrease in value (impairment loss)                                                                       4 000

Journals: 30 June 20X4                                                                  Debit          Credit
Impairment loss (expense)                                                                     4 000
 - Plant: accumulated depreciation and impairment losses                                                  4 000
Re-measurement of non-current asset held for sale: decrease in fair value
less costs to sell


4.4    Reversal of classification as ‘held for sale’ (IFRS 5.26 - .29)

If a non-current asset that was previously classified as ‘held for sale’ no longer meets the
criteria necessary for such a classification, the asset must immediately cease to be classified
as ‘held for sale’ and must be re-measured to the lower of:
• its carrying amount had the non-current asset never been classified as ‘held for
     sale’(adjusted for any depreciation, amortisation and/ or revaluations that would have
     been recognised had the asset not been classified as held for sale); and
• its recoverable amount.




                                                   321                                                Chapter 9
Gripping IFRS                              Non-current assets held for sale and discontinued operations



Example 6: re-measurement of assets no longer classified as ‘held for sale’

Plant, with a cost of C100 000 (1 January 20X1) and accumulated depreciation of C20 000 on
31 December 20X2 (10% straight-line for 2 years), was reclassified as ‘held for sale’ on
31 December 20X2 and immediately impaired to its ‘fair value less costs to sell’ of C65 000.
On 30 June 20X3 (six months later), it ceased to meet all criteria necessary for classification
as ‘held for sale’. On this date its recoverable amount is determined to be C85 000.

Required:
Show all journal entries relating to the re-measurement of plant previously held as a ‘non-
current asset held for sale’.

Solution to example 6: re-measurement of assets no longer classified as ‘held for sale’

Workings:                                                                                     C
New carrying amount (30 June 20X3) to be lower of:                                            75 000
• Carrying amount had the asset never     100 000 – 20 000 – 100 000 x 10% x 6/12             75 000
    been classified as ‘held for sale’
• Recoverable amount                      Given                                                85 000
Current carrying amount (30 June 20X3)    Fair value – costs to sell                          (65 000)
Impairment loss to be reversed                                                                 10 000

Journals:                                                                     Debit          Credit

30 June 20X3
Plant: accumulated impairment loss                                              10 000
 - Impairment loss reversed (income)                                                            10 000
Reversal of impairment loss on reclassification of ‘non-current asset
held for sale’ as ‘property, plant and equipment’: criteria no longer met

Note: Depreciation on this asset will now begin again.



5.    Non-current assets held for sale: disclosure (IFRS 5.30 and .38 - .42)

5.1   Overview

Extra disclosure is required where the financial statements include either:
• a ‘non-current asset held for sale’; or
• a ‘sale of a non-current asset’.

The classification affects the period during which it was classified as ‘held for sale’. This
means that no adjustment should be made to the measurement or presentation of the affected
assets in the comparative periods presented.

5.2   In the statement of financial position

Non-current assets (or non-current assets within a disposal group) that are ‘held for sale’ must
be shown separately in the statement of financial position.

If a disposal group includes liabilities, these liabilities must also be shown separately from
other liabilities in the statement of financial position and may not be set-off against the assets
within the disposal group.




                                                  322                                       Chapter 9
Gripping IFRS                          Non-current assets held for sale and discontinued operations



5.3   In the statement of financial position or notes thereto

Major classifications of assets within the total of the ‘non-current assets held for sale’ and
major classifications of liabilities within the total ‘liabilities of a disposal group’ must be
shown in the notes (unless shown in the statement of financial position).

5.4   Other note disclosure

An entity shall disclose the following information in the notes in the period in which a non-
current asset (or disposal group) has been classified as held for sale or sold:
a) a description of the non-current asset (or disposal group);
b) a description of the facts and circumstances of the sale, or leading to the expected
    disposal, and the expected manner and timing of that disposal;
c) the gain or loss recognised in accordance with IFRS 5 (paragraph 20-22) and, if not
    separately presented in the statement of comprehensive income, the caption in the
    statement of comprehensive income that includes that gain or loss;
d) if applicable, the segment in which the non-current asset (or disposal group) is presented
    in accordance with IAS 14 Segment Reporting.

If, during the current period, there was a decision to reverse the plan to sell the non-current
asset (or disposal group), the following extra disclosure would be required:
a) the description of the facts and circumstances leading to the decision not to sell; and
b) the effect of the decision on the results of operations for all periods presented.

Example 7: disclosure of non-current assets held for sale

Assume that an entity owns only the following non-current assets:
• Plant; and
• Factory buildings.

Details of the plant are as follows:
• Plant was purchased on 1 January 20X1 at a cost of C100 000;
• Depreciation is provided over 10 years to a nil residual value on the straight-line basis;
• Plant was reclassified as ‘held for sale’ on 31 December 20X2 and immediately impaired
   to its ‘fair value less costs to sell’ of C65 000;
• On 30 June 20X3 (six months later), plant ceased to meet all criteria necessary for
   classification as ‘held for sale’, on which date its recoverable amount is C85 000.

Details of the factory buildings are as follows:
• The factory buildings were purchased on 1 January 20X1 at a cost of C600 000,
• Depreciation is provided over 10 years to nil residual values on the straight-line basis
• Factory buildings were reclassified as ‘held for sale’ on 30 June 20X3’ at a ‘fair value
   less cost to sell’ of C445 000.

Required:
Disclose all information necessary in relation to the plant and factory buildings in the
financial statements for the year ended 31 December 20X3.




                                             323                                        Chapter 9
Gripping IFRS                               Non-current assets held for sale and discontinued operations



Solution to example 7: disclosure of non-current assets held for sale

Comment: this example explains how to disclose non-current assets held for sale, as well as how to
disclose a non-current asset that is no longer held for sale.

Company name
Statement of financial position
At 31 December 20X3
                                                                                    20X3            20X2
Non-current assets                                                                    C               C
Property, plant and equipment                                      26                70 000         480 000
Non-current assets (and disposal groups) held for sale             27               445 000          65 000
Non-current liabilities
Liabilities of a disposal group (for disclosure purposes only)     27                    xxx            xxx

Company name
Notes to the financial statements
For the year ended 31 December 20X3
                                                                                   20X3             20X2
                                                                                     C               C
5. Profit before tax
    Profit before tax is stated after taking into consideration the following (income)/ expenses:
     Depreciation – factory building                                               30 000            60 000
     Depreciation – plant                                                           5 000            10 000
     Impairment loss – asset held for sale                                          5 000            15 000
     Impairment loss reversed – asset no longer held for sale                     (10 000)                0
26. Property, plant and equipment
    Factory building                                                                    0           480 000
    Plant                                                                          70 000                 0
                                                                                   70 000           480 000
    Factory building:
    Net carrying amount – 1 January                                               480 000           540 000
    Gross carrying amount – 1 January                                             600 000           600 000
    Accumulated depreciation and impairment losses – 1 January                   (120 000)          (60 000)
    Depreciation (to 30 June 20X5)                                                (30 000)          (60 000)
    Impairment loss (to fair value less costs to sell: 450 000 – 445 000)          (5 000)                0
    Non-current asset now classified as ‘held for sale’                          (445 000)                0
    Net carrying amount – 31 December                                                    0       480 000
    Gross carrying amount – 31 December                                                  0       600 000
    Accumulated depreciation and impairment losses – 31 December                         0      (120 000)
    Plant:
    Net carrying amount – 1 January                                                      0            90 000
    Gross carrying amount – 1 January                                                    0          100 000
    Accumulated depreciation and impairment losses – 1 January                           0           (10 000)
    Non-current asset no longer classified as ‘held for sale’                      65 000                 0
    Reversal of impairment loss (to lower of HCA: 75 000 or RA:85 000)             10 000                 0
    Depreciation (20X3: 75 000 / 7,5 remaining years x 6/12)                        (5 000)         (10 000)
    Impairment loss (to fair value less costs to sell: 80 000 – 65 000)                  0          (15 000)
    Non-current asset now classified as ‘held for sale’                                  0          (65 000)
    Net carrying amount – 31 December                                              70 000                  0
    Gross carrying amount – 31 December                                           100 000                  0
    Accumulated depreciation and impairment losses – 31 December                  (30 000)                 0


                                                   324                                         Chapter 9
Gripping IFRS                               Non-current assets held for sale and discontinued operations



27. Non-current assets held for sale                                            20X3              20X2
                                                                                  C                C
    Factory buildings                                                          445 000              0
    Plant                                                                         0              65 000
    Less non-current interest bearing liabilities (disclosure purpose)            0                 0
                                                                               445 000           65 000
    The company is transferring its business to a new location and thus the existing factory building is
    to be sold (circumstances leading to the decision).
    The sale is expected to take place within 7 months of the end of the reporting period (expected
    timing). The factory building is expected to be sold as a going concern (expected manner of sale).
    The plant is no longer classified as ‘held for sale’ since it is now intended to be redeployed to other
    existing factories rather than to be sold together with the factory buildings (reasons for the
    decision not to sell).
         The effect on current year profit from operations is as follows:               C
         - Gross (Impairment loss reversed: 10 000 – deprec.:5 000)                     5 000
         - Tax                                                                         (1 500)
         - Net                                                                          3 500



6. Discontinued operations: identification (IAS 5.31 - .36)

IFRS 5 requires that, where a component is identified as a discontinued operation, it must be
separately disclosed in the financial statements. The following definitions are provided in
IFRS 5:

A component of an entity comprise:
• operations and cash flows
• that can be clearly distinguished, operationally and for financial reporting purposes,
• from the rest of the entity.

A component of an entity may be a cash-generating unit or any group thereof.

A discontinued operation is
• a component of an entity that has either been
   - disposed of, or
   - is classified as held for sale;
• and meets one of the following criteria:
   - represents a separate major line of business or geographical area of operations; or
   - is part of a single co-ordinated plan to dispose of a separate major line of business or
       geographical area of operations; or
   - is a subsidiary acquired exclusively with a view to resale.


7. Discontinued operations: measurement

A discontinued operation is, in effect, constituted by non-current assets (or disposal groups)
held for sale that, together, comprise a component that meets the definition of a ‘discontinued
operation’. Therefore, the principles that are adopted when measuring the individual non-
current assets (or disposal groups) held for sale are also used when measuring the elements of
a discontinued operation.

If the non-current asset (or disposal group) does not meet the definition of a ‘component’, the
related transactions and adjustments will not be disclosed as ‘discontinued operations’ but
rather as part of ‘continuing operations’.


                                                  325                                            Chapter 9
Gripping IFRS                                    Non-current assets held for sale and discontinued operations



8. Discontinued operations: disclosure

8.1     In the statement of comprehensive income
A single amount must be presented on the face of the statement of comprehensive income
being the total of:
• the post-tax profit or loss of the discontinued operations;
• the post-tax gain or loss recognised on measurement to fair value less costs to sell; and
• the post-tax gain or loss recognised on disposal of assets/ disposal groups making up the
    discontinued operations.
An analysis of this single amount that is presented in the statement of comprehensive income
must be presented ‘for all periods presented’. This analysis may be done in the statement of
comprehensive income (see suggested presentation option A on the next page) or in the notes
(see suggested presentation option B on the next page) and must show the following:
• revenue of discontinued operations;
• expenses of discontinued operations;
• profit (or loss) before tax of discontinued operations; and
• tax expense of discontinued operations.
An entity must also disclose the following either in the statement of comprehensive income or
in the notes thereto ‘for all periods presented’ (with the exception of the change in estimate):
• gain or loss on re-measurement to ‘fair value less selling costs’;
• gain or loss on disposal of the discontinued operation (made up by assets/ disposal
     groups);
• tax effects of the above; and
• changes to estimates made in respect of discontinued operations disposed of in a prior
     period (showing nature and amount); examples of such changes include outcomes of
     previous uncertainties relating to:
     - the disposal transaction (e.g. adjustments to the selling price); and
     - the operations of the component before its disposal (e.g. adjustments to warranty/
         legal obligations retained by the entity).
Option A: If the analysis of the profit or loss is presented on the face of the statement of
comprehensive income, the statement of comprehensive income will look something like this
(the figures are all assumed):

Example Ltd
Statement of comprehensive income
For the year ended 31 December 20X3 (extracts)
                                     20X3          20X3        20X3        20X2          20X2          20X2
                                     C’000         C’000       C’000       C’000         C’000         C’000
                                   Continuing   Discontinued   Total     Continuing   Discontinued     Total
Revenue                                  800            150                    800            790
Expenses                                (300)         (100)                   (400)         (500)
Profit before tax                       500             50                    400            290
Taxation expense                       (150)           (60)                  (180)           (97)
Gains/ (losses) after tax                               40                                     7
Gain/ (loss): re-measurement                            30                                    10
to fair value less costs to sell
Gain/ (loss): disposal of assets                        20                                     0
in the discontinued operations
Tax on gains/ (losses)                                 (10)                                   (3)

Profit for the period                   350             30      380           220            200        420
Other comprehensive income                0              0        0             0              0          0
Total comprehensive income              350             30      380           220            200        420



                                                        326                                          Chapter 9
Gripping IFRS                              Non-current assets held for sale and discontinued operations



Option B: If the total profit or loss is presented in the statement of comprehensive income,
with the analysis in the notes, the statement of comprehensive income and notes will look
something like this (the figures are all assumed):

Example Ltd
Statement of comprehensive income
For the year ended 31 December 20X3 (extracts)
                                                                              20X3              20X2
                                                                 Note         C’000             C’000
Revenue                                                                         800               800
Expenses                                                                       (300)             (400)
Profit before tax                                                               500               400
Taxation expense                                                               (150)             (180)
Profit from continuing operations                                               350               220
Profit from discontinued operations                              4&5             30               200
Profit for the period                                                           380               420
Other comprehensive income                                                        0                 0
Total comprehensive income                                                      380               420

Example Ltd
Notes to the financial statements
For the year ended 31 December 20X3 (extracts)
                                                                              20X3              20X2
4. Discontinued operation: analysis of profit                                 C’000             C’000
The profit from discontinued operations is analysed as follows:
    • Revenue                                                                    150               790
    • Expenses                                                                  (100)             (500)
    • Profit before tax                                                           50               290
    • Tax                                                                       (60)              (97)
    • Gains/ (losses) after tax                                                   40                 7
    • Gain/ (loss on re-measurement to fair value less costs to sell              30                10
    • Gain/ (loss) on disposal of assets in the/ the discontinued                 20                 0
        operations
    • Tax on gains/ (losses)                                                     (10)               (3)
    • Profit for the period                                                        30              200

8.2   In the statement of cash flows

In respect of discontinued operations, an entity shall disclose the following either on the face
of the statement of cash flows or in the notes thereto ‘for all periods presented’ [para 33(c)]:
• net cash flows from operating activities;
• net cash flows from investing activities; and
• net cash flows from financing activities.

Example Ltd
Notes to the statement of cash flows
For the year ended 31 December 20X3 (extracts)
                                                                                    20X3         20X2
4. Discontinued operation                                                          C’000         C’000
    Included in the statement of cash flows are the following net cash flows resulting from a
    discontinued operation:
           Net cash flows from operating activities (assumed figures)                  5             6
           Net cash flows from investing activities (assumed figures)                  0             1
           Net cash flows from financing activities (assumed figures)                 (8)           (4)
           Net cash outflows (assumed figures)                                        (3)            3



                                                  327                                           Chapter 9
Gripping IFRS                                Non-current assets held for sale and discontinued operations



8.3      Other note disclosure

8.3.1 Components no longer held for sale (IFRS 5.37)

Where the component is no longer ‘held for sale’, the amounts previously disclosed as
‘discontinued operations’ in the prior periods must be reclassified and included in ‘continuing
operations’. This will facilitate better comparability.

See the examples of disclosure provided in 8.1 and assume that the discontinued operation
was first classified as such in 20X2, but that during 20X3 the criteria for classification as
‘discontinued’ were no longer met. Notice that the 20X2 figures shown below, whereas
previously split into ‘continuing’, ‘discontinuing’ and ‘total’ (in 8.1) are now restated in one
column. Although IFRS 5 does not require it, it is suggested that a note be included
explaining to the user that a previously classified ‘discontinued operation’ has been
reabsorbed into the figures representing the ‘continuing operations’ of the entity, thus
explaining the re-presentation of the 20X2 figures.

Example Ltd
Statement of comprehensive income
For the year ended 31 December 20X3 (extracts)
                                                                                   20X3          20X2
                                                                                   C’000         C’000
                                                                                                Restated
Revenue                                                                              1 000          1 600
Expenses                                                                           (400)          (900)
Profit before tax                                                                   600            700
Tax expense                                                                        (220)          (280)
Profit for the period                                                               380            420
Other comprehensive income                                                            0               0
Total comprehensive income                                                          380            420

The above amounts are assumed amounts: notice how they tie up with the previous explanatory
examples in Option A and Option B.

8.3.2 Criteria met after the end of the reporting period (IAS 5.12)

If the criteria for separate classification and measurement as ‘held for sale’ are met during the
post-reporting date period, no adjustments should be made to the amounts and no
reclassification of the assets as ‘held for sale’ should take place. This is treated as a non-
adjusting event with the following disclosure being necessary:
• a description of the non-current asset (or disposal group);
• a description of the facts and circumstances leading to the expected disposal;
• the expected manner and timing of the disposal; and
• the segment (if applicable) in which the non-current asset (or disposal group) is presented.

The note disclosure of an event after the reporting period might look like this:

Example Ltd
Notes to the financial statements
For the year ended 31 December 20X3 (extracts)

4. Events after the reporting period

      On 15 February 20X4, the board of directors decided to dispose of the shoe division following
      severe losses incurred by it during the past 2 years. The division is expected to continue operations
      until 30 April 20X4, after which its assets will be sold on a piecemeal basis. The entire disposal of
      the division is expected to be completed by 31 August 20X4.


                                                    328                                        Chapter 9
Gripping IFRS                         Non-current assets held for sale and discontinued operations



                                       9. Summary



                                                IFRS 5



              Non-current assets                                       Discontinued
                 held for sale                                          operations




                            Non-current assets held for sale


                                         Identification



          General                  If asset not expected               Assets acquired with
                                    to be sold within 1 yr                intention to sell
Normal 6 criteria                  3 scenarios and                   2 criteria
                                   related criteria



                                       Measurement



          Cost model                     Revaluation model                Assets acquired with
                                                                            intention to sell
Initially:                         Initially:                           Initially:

at cost                            at cost                              Lower of CA (cost) and
                                                                        FV – costs to sell
Subsequently:                      Subsequently:

Before reclassification:           Before reclassification:
Depreciate and impair              Depreciate; revalue and
                                   impair

When reclassifying:                When reclassifying:
Remeasure on                       Remeasure on revaluation
cost model                         model

Adjust to lower of CA or           Adjust to lower of CA or FV
FV – CtS                           – CtS

Stop depreciating                  Stop depreciating

Transfer to NCAHforS               Transfer to NCAHforS

Remeasure to latest FV –           Remeasure to latest FV –
CtS                                CtS
(reversals of IL limited           (reversals of IL limited to
to accumulated IL’s)               accumulated IL’s)




                                             329                                       Chapter 9
Gripping IFRS                      Non-current assets held for sale and discontinued operations



                         Non-current assets held for sale


                                No longer held for sale




         Transfer back to PPE                    Remeasure to lower of:
                                                 • CA (had asset never been classified
                                                    as NCAHforS); and
                                                 • RA
                                                 Resume depreciation




                                   Discontinued operations



     Identification                 Measurement                           Disclosure


A component that has            Same as for non-                  Statement of
been disposed of or is          current assets held               comprehensive income:
classified as held for          for sale                          Face:
sale and is:                                                      Total profit or loss from
• Separate major line                                             discontinued operations
    or area                                                       Notes or on the face:;
• Part of a single                                                Analysis of total profit
    disposal plan or                                              Gain or loss on re-
• Is a subsidiary                                                 measurement
    acquired to sell                                              Gain or loss on disposal of
                                                                  assets
                                                                  Tax effects of above
                                                                  Changes in estimates
                                                                  Statement of cash flows:
                                                                  (face or notes)
                                                                  Operating activities
                                                                  Investing activities
                                                                  Financing activities
                                                                  Other notes:
                                                                  Components no longer held
                                                                  for sale
                                                                  Criteria met after the end
                                                                  of the reporting period




                                         330                                        Chapter 9

Weitere ähnliche Inhalte

Was ist angesagt?

Risk and Return
Risk and ReturnRisk and Return
Risk and Returnsaadiakh
 
WORKING CAPITAL MANAGEMENT
WORKING CAPITAL MANAGEMENTWORKING CAPITAL MANAGEMENT
WORKING CAPITAL MANAGEMENTipermeeta
 
Financial market and institutions
Financial market and institutionsFinancial market and institutions
Financial market and institutionsHung Tran
 
Ind AS 40 : Investment Property
Ind AS 40 : Investment PropertyInd AS 40 : Investment Property
Ind AS 40 : Investment PropertyCA Aman Agrawal
 
Topic 4 employee_benefit_a132_1_
Topic 4 employee_benefit_a132_1_Topic 4 employee_benefit_a132_1_
Topic 4 employee_benefit_a132_1_kim rae KI
 
working capital management 1
working capital management 1working capital management 1
working capital management 1PaRth PaTel
 
Presentation on dividend policy
Presentation on dividend policyPresentation on dividend policy
Presentation on dividend policyStudent
 
Valuation of investments
Valuation of investmentsValuation of investments
Valuation of investmentsKapil Chhabra
 
Financial Instruments
Financial InstrumentsFinancial Instruments
Financial InstrumentsAziz Zaman
 
03 chapter 4 deductions from gross estate part 02
03 chapter 4 deductions from gross estate part 0203 chapter 4 deductions from gross estate part 02
03 chapter 4 deductions from gross estate part 02Flab Villasencio
 
Financial markets and institutions ITM3
Financial markets and institutions ITM3Financial markets and institutions ITM3
Financial markets and institutions ITM3Aram Mohammed
 
Capital budgeting techniques
Capital budgeting techniquesCapital budgeting techniques
Capital budgeting techniquesVJTI Production
 
Working capital management
Working capital managementWorking capital management
Working capital managementankita3590
 
Intangible Assets IAS 38
Intangible Assets IAS 38Intangible Assets IAS 38
Intangible Assets IAS 38serveuuu
 
Issuing securities to the public
Issuing securities to the publicIssuing securities to the public
Issuing securities to the publicOnline
 

Was ist angesagt? (20)

Risk and Return
Risk and ReturnRisk and Return
Risk and Return
 
Ppt ch01
Ppt ch01Ppt ch01
Ppt ch01
 
WORKING CAPITAL MANAGEMENT
WORKING CAPITAL MANAGEMENTWORKING CAPITAL MANAGEMENT
WORKING CAPITAL MANAGEMENT
 
Financial market and institutions
Financial market and institutionsFinancial market and institutions
Financial market and institutions
 
Ind AS 40 : Investment Property
Ind AS 40 : Investment PropertyInd AS 40 : Investment Property
Ind AS 40 : Investment Property
 
Topic 4 employee_benefit_a132_1_
Topic 4 employee_benefit_a132_1_Topic 4 employee_benefit_a132_1_
Topic 4 employee_benefit_a132_1_
 
working capital management 1
working capital management 1working capital management 1
working capital management 1
 
Presentation on dividend policy
Presentation on dividend policyPresentation on dividend policy
Presentation on dividend policy
 
Unit 3 hybrid securities
Unit 3   hybrid securitiesUnit 3   hybrid securities
Unit 3 hybrid securities
 
Valuation of investments
Valuation of investmentsValuation of investments
Valuation of investments
 
Financial Instruments
Financial InstrumentsFinancial Instruments
Financial Instruments
 
03 chapter 4 deductions from gross estate part 02
03 chapter 4 deductions from gross estate part 0203 chapter 4 deductions from gross estate part 02
03 chapter 4 deductions from gross estate part 02
 
Chapter 14
Chapter 14Chapter 14
Chapter 14
 
Financial markets and institutions ITM3
Financial markets and institutions ITM3Financial markets and institutions ITM3
Financial markets and institutions ITM3
 
Capital budgeting techniques
Capital budgeting techniquesCapital budgeting techniques
Capital budgeting techniques
 
Working capital management
Working capital managementWorking capital management
Working capital management
 
Intangible Assets IAS 38
Intangible Assets IAS 38Intangible Assets IAS 38
Intangible Assets IAS 38
 
Issuing securities to the public
Issuing securities to the publicIssuing securities to the public
Issuing securities to the public
 
Equity share
Equity shareEquity share
Equity share
 
Ifrs 2
Ifrs 2Ifrs 2
Ifrs 2
 

Andere mochten auch

solusi manual advanced acc zy Chap006
solusi manual advanced acc zy Chap006solusi manual advanced acc zy Chap006
solusi manual advanced acc zy Chap006Suzie Lestari
 
Non current assets held for sale and discontinued
Non current assets held for sale and discontinuedNon current assets held for sale and discontinued
Non current assets held for sale and discontinuedHyderabad Chapter of ICWAI
 
Chapter12 governmentgrants2008
Chapter12  governmentgrants2008Chapter12  governmentgrants2008
Chapter12 governmentgrants2008Sajid Ali
 
Chapter21 financialinstruments2007
Chapter21 financialinstruments2007Chapter21 financialinstruments2007
Chapter21 financialinstruments2007Sajid Ali
 
Chapter3 deferred tax2008
Chapter3 deferred tax2008Chapter3 deferred tax2008
Chapter3 deferred tax2008Sajid Ali
 
Chapter25 financialanalysis2008
Chapter25 financialanalysis2008Chapter25 financialanalysis2008
Chapter25 financialanalysis2008Sajid Ali
 
Aset tak lancar yg dimiliki u dijual
Aset tak lancar yg dimiliki u dijualAset tak lancar yg dimiliki u dijual
Aset tak lancar yg dimiliki u dijualUchiha Emzhie
 
Psak 58-aset-tidak-lancar-yang-dimiliki-untuk-dijual-dan-operasi-yang-dihenti...
Psak 58-aset-tidak-lancar-yang-dimiliki-untuk-dijual-dan-operasi-yang-dihenti...Psak 58-aset-tidak-lancar-yang-dimiliki-untuk-dijual-dan-operasi-yang-dihenti...
Psak 58-aset-tidak-lancar-yang-dimiliki-untuk-dijual-dan-operasi-yang-dihenti...Sri Apriyanti Husain
 

Andere mochten auch (9)

solusi manual advanced acc zy Chap006
solusi manual advanced acc zy Chap006solusi manual advanced acc zy Chap006
solusi manual advanced acc zy Chap006
 
Non current assets held for sale and discontinued
Non current assets held for sale and discontinuedNon current assets held for sale and discontinued
Non current assets held for sale and discontinued
 
Chapter12 governmentgrants2008
Chapter12  governmentgrants2008Chapter12  governmentgrants2008
Chapter12 governmentgrants2008
 
Chapter21 financialinstruments2007
Chapter21 financialinstruments2007Chapter21 financialinstruments2007
Chapter21 financialinstruments2007
 
Chapter3 deferred tax2008
Chapter3 deferred tax2008Chapter3 deferred tax2008
Chapter3 deferred tax2008
 
Chapter25 financialanalysis2008
Chapter25 financialanalysis2008Chapter25 financialanalysis2008
Chapter25 financialanalysis2008
 
Aset tak lancar yg dimiliki u dijual
Aset tak lancar yg dimiliki u dijualAset tak lancar yg dimiliki u dijual
Aset tak lancar yg dimiliki u dijual
 
Ifrs 5 case studies q & a
Ifrs 5 case studies q & aIfrs 5 case studies q & a
Ifrs 5 case studies q & a
 
Psak 58-aset-tidak-lancar-yang-dimiliki-untuk-dijual-dan-operasi-yang-dihenti...
Psak 58-aset-tidak-lancar-yang-dimiliki-untuk-dijual-dan-operasi-yang-dihenti...Psak 58-aset-tidak-lancar-yang-dimiliki-untuk-dijual-dan-operasi-yang-dihenti...
Psak 58-aset-tidak-lancar-yang-dimiliki-untuk-dijual-dan-operasi-yang-dihenti...
 

Ähnlich wie Chapter9 noncurrentassetsheldforsale2008

Ifrs An Overview 2011
Ifrs An Overview 2011Ifrs An Overview 2011
Ifrs An Overview 2011lvercammen
 
IFRS An Overview 2011
IFRS An Overview 2011IFRS An Overview 2011
IFRS An Overview 2011fmathot
 
IFRS An Overview 2011
IFRS An Overview 2011IFRS An Overview 2011
IFRS An Overview 2011SvenSmeulders
 
Ifrs An Overview 2011
Ifrs An Overview 2011Ifrs An Overview 2011
Ifrs An Overview 2011lverrijssen
 
Ifrs An Overview 2011
Ifrs An Overview 2011Ifrs An Overview 2011
Ifrs An Overview 2011rvanelsen
 
IFRS An Overview 2011
IFRS An Overview 2011IFRS An Overview 2011
IFRS An Overview 2011adebacker
 
Ifrs An Overview 2011
Ifrs An Overview 2011Ifrs An Overview 2011
Ifrs An Overview 2011mdeschrijver
 
cit 10QType set Final
cit 10QType set Finalcit 10QType set Final
cit 10QType set Finalfinance28
 
Chap 6 - IAS 38 - Intangible Assets.pptx
Chap 6 - IAS 38 - Intangible Assets.pptxChap 6 - IAS 38 - Intangible Assets.pptx
Chap 6 - IAS 38 - Intangible Assets.pptxKashif Butt
 
cit 3Q08%2010Q
cit 3Q08%2010Qcit 3Q08%2010Q
cit 3Q08%2010Qfinance28
 
cit 3Q 08%2010Q
cit 3Q 08%2010Qcit 3Q 08%2010Q
cit 3Q 08%2010Qfinance28
 
CIT_2007_Q1_10-Q
CIT_2007_Q1_10-QCIT_2007_Q1_10-Q
CIT_2007_Q1_10-Qfinance28
 
cit 10QTypesetFinal
cit 10QTypesetFinalcit 10QTypesetFinal
cit 10QTypesetFinalfinance28
 
2007%20Q2%20-%2010-Q
2007%20Q2%20-%2010-Q2007%20Q2%20-%2010-Q
2007%20Q2%20-%2010-Qfinance28
 
CIT_2007_Q1_10-Q
CIT_2007_Q1_10-QCIT_2007_Q1_10-Q
CIT_2007_Q1_10-Qfinance28
 
international paper Q1 2005 10-Q
international paper Q1 2005 10-Qinternational paper Q1 2005 10-Q
international paper Q1 2005 10-Qfinance12
 

Ähnlich wie Chapter9 noncurrentassetsheldforsale2008 (20)

Ifrs An Overview 2011
Ifrs An Overview 2011Ifrs An Overview 2011
Ifrs An Overview 2011
 
Ifrs An Overview 2011
Ifrs An Overview 2011Ifrs An Overview 2011
Ifrs An Overview 2011
 
IFRS An Overview 2011
IFRS An Overview 2011IFRS An Overview 2011
IFRS An Overview 2011
 
IFRS An Overview 2011
IFRS An Overview 2011IFRS An Overview 2011
IFRS An Overview 2011
 
Ifrs An Overview 2011
Ifrs An Overview 2011Ifrs An Overview 2011
Ifrs An Overview 2011
 
Ifrs An Overview 2011
Ifrs An Overview 2011Ifrs An Overview 2011
Ifrs An Overview 2011
 
IFRS An Overview 2011
IFRS An Overview 2011IFRS An Overview 2011
IFRS An Overview 2011
 
Ifrs An Overview 2011
Ifrs An Overview 2011Ifrs An Overview 2011
Ifrs An Overview 2011
 
cit 10QType set Final
cit 10QType set Finalcit 10QType set Final
cit 10QType set Final
 
Chap 6 - IAS 38 - Intangible Assets.pptx
Chap 6 - IAS 38 - Intangible Assets.pptxChap 6 - IAS 38 - Intangible Assets.pptx
Chap 6 - IAS 38 - Intangible Assets.pptx
 
Ias38
Ias38Ias38
Ias38
 
cit 3Q08%2010Q
cit 3Q08%2010Qcit 3Q08%2010Q
cit 3Q08%2010Q
 
cit 3Q 08%2010Q
cit 3Q 08%2010Qcit 3Q 08%2010Q
cit 3Q 08%2010Q
 
CIT_2007_Q1_10-Q
CIT_2007_Q1_10-QCIT_2007_Q1_10-Q
CIT_2007_Q1_10-Q
 
Financial accounting icab chapter 6 intangible assets
Financial accounting icab chapter 6 intangible assetsFinancial accounting icab chapter 6 intangible assets
Financial accounting icab chapter 6 intangible assets
 
CIT%2010Q
CIT%2010QCIT%2010Q
CIT%2010Q
 
cit 10QTypesetFinal
cit 10QTypesetFinalcit 10QTypesetFinal
cit 10QTypesetFinal
 
2007%20Q2%20-%2010-Q
2007%20Q2%20-%2010-Q2007%20Q2%20-%2010-Q
2007%20Q2%20-%2010-Q
 
CIT_2007_Q1_10-Q
CIT_2007_Q1_10-QCIT_2007_Q1_10-Q
CIT_2007_Q1_10-Q
 
international paper Q1 2005 10-Q
international paper Q1 2005 10-Qinternational paper Q1 2005 10-Q
international paper Q1 2005 10-Q
 

Mehr von Sajid Ali

Chapter24 cashflowstatements2008
Chapter24 cashflowstatements2008Chapter24 cashflowstatements2008
Chapter24 cashflowstatements2008Sajid Ali
 
Chapter23 earningspershare2008
Chapter23 earningspershare2008Chapter23 earningspershare2008
Chapter23 earningspershare2008Sajid Ali
 
Chapter22 sharecapital2008
Chapter22  sharecapital2008Chapter22  sharecapital2008
Chapter22 sharecapital2008Sajid Ali
 
Chapter20 forwardexchangecontracts2008
Chapter20 forwardexchangecontracts2008Chapter20 forwardexchangecontracts2008
Chapter20 forwardexchangecontracts2008Sajid Ali
 
Chapter19 foreigncurrencytransactions2008
Chapter19 foreigncurrencytransactions2008Chapter19 foreigncurrencytransactions2008
Chapter19 foreigncurrencytransactions2008Sajid Ali
 
Chapter18 policiesestimatesanderrors2008
Chapter18  policiesestimatesanderrors2008Chapter18  policiesestimatesanderrors2008
Chapter18 policiesestimatesanderrors2008Sajid Ali
 
Chapter17 provisionsandpostbalancesheetevents2008
Chapter17  provisionsandpostbalancesheetevents2008Chapter17  provisionsandpostbalancesheetevents2008
Chapter17 provisionsandpostbalancesheetevents2008Sajid Ali
 
Chapter16 employeebenefits2008
Chapter16 employeebenefits2008Chapter16 employeebenefits2008
Chapter16 employeebenefits2008Sajid Ali
 
Chapter15 revenue2008
Chapter15 revenue2008Chapter15 revenue2008
Chapter15 revenue2008Sajid Ali
 
Chapter14 leases lessors2008
Chapter14  leases lessors2008Chapter14  leases lessors2008
Chapter14 leases lessors2008Sajid Ali
 
Chapter13 leaseslessees2008
Chapter13 leaseslessees2008Chapter13 leaseslessees2008
Chapter13 leaseslessees2008Sajid Ali
 
Chapter11 borrowingcosts2008
Chapter11  borrowingcosts2008Chapter11  borrowingcosts2008
Chapter11 borrowingcosts2008Sajid Ali
 
Chapter10 impairmentofassets2008
Chapter10  impairmentofassets2008Chapter10  impairmentofassets2008
Chapter10 impairmentofassets2008Sajid Ali
 
Chapter8 investmentproperties 2008
Chapter8  investmentproperties 2008Chapter8  investmentproperties 2008
Chapter8 investmentproperties 2008Sajid Ali
 
Chapter7 intangibleassets2008
Chapter7 intangibleassets2008Chapter7 intangibleassets2008
Chapter7 intangibleassets2008Sajid Ali
 
Chapter6 propertyplantandequipmentmodels2008
Chapter6 propertyplantandequipmentmodels2008Chapter6 propertyplantandequipmentmodels2008
Chapter6 propertyplantandequipmentmodels2008Sajid Ali
 
Chapter5 propertyplantandequipmentbasics2008
Chapter5 propertyplantandequipmentbasics2008Chapter5 propertyplantandequipmentbasics2008
Chapter5 propertyplantandequipmentbasics2008Sajid Ali
 
Chapter4 inventory2008
Chapter4 inventory2008Chapter4 inventory2008
Chapter4 inventory2008Sajid Ali
 
Chapter2 taxation2008
Chapter2 taxation2008Chapter2 taxation2008
Chapter2 taxation2008Sajid Ali
 
Chapter1 the pillars_2008
Chapter1 the pillars_2008Chapter1 the pillars_2008
Chapter1 the pillars_2008Sajid Ali
 

Mehr von Sajid Ali (20)

Chapter24 cashflowstatements2008
Chapter24 cashflowstatements2008Chapter24 cashflowstatements2008
Chapter24 cashflowstatements2008
 
Chapter23 earningspershare2008
Chapter23 earningspershare2008Chapter23 earningspershare2008
Chapter23 earningspershare2008
 
Chapter22 sharecapital2008
Chapter22  sharecapital2008Chapter22  sharecapital2008
Chapter22 sharecapital2008
 
Chapter20 forwardexchangecontracts2008
Chapter20 forwardexchangecontracts2008Chapter20 forwardexchangecontracts2008
Chapter20 forwardexchangecontracts2008
 
Chapter19 foreigncurrencytransactions2008
Chapter19 foreigncurrencytransactions2008Chapter19 foreigncurrencytransactions2008
Chapter19 foreigncurrencytransactions2008
 
Chapter18 policiesestimatesanderrors2008
Chapter18  policiesestimatesanderrors2008Chapter18  policiesestimatesanderrors2008
Chapter18 policiesestimatesanderrors2008
 
Chapter17 provisionsandpostbalancesheetevents2008
Chapter17  provisionsandpostbalancesheetevents2008Chapter17  provisionsandpostbalancesheetevents2008
Chapter17 provisionsandpostbalancesheetevents2008
 
Chapter16 employeebenefits2008
Chapter16 employeebenefits2008Chapter16 employeebenefits2008
Chapter16 employeebenefits2008
 
Chapter15 revenue2008
Chapter15 revenue2008Chapter15 revenue2008
Chapter15 revenue2008
 
Chapter14 leases lessors2008
Chapter14  leases lessors2008Chapter14  leases lessors2008
Chapter14 leases lessors2008
 
Chapter13 leaseslessees2008
Chapter13 leaseslessees2008Chapter13 leaseslessees2008
Chapter13 leaseslessees2008
 
Chapter11 borrowingcosts2008
Chapter11  borrowingcosts2008Chapter11  borrowingcosts2008
Chapter11 borrowingcosts2008
 
Chapter10 impairmentofassets2008
Chapter10  impairmentofassets2008Chapter10  impairmentofassets2008
Chapter10 impairmentofassets2008
 
Chapter8 investmentproperties 2008
Chapter8  investmentproperties 2008Chapter8  investmentproperties 2008
Chapter8 investmentproperties 2008
 
Chapter7 intangibleassets2008
Chapter7 intangibleassets2008Chapter7 intangibleassets2008
Chapter7 intangibleassets2008
 
Chapter6 propertyplantandequipmentmodels2008
Chapter6 propertyplantandequipmentmodels2008Chapter6 propertyplantandequipmentmodels2008
Chapter6 propertyplantandequipmentmodels2008
 
Chapter5 propertyplantandequipmentbasics2008
Chapter5 propertyplantandequipmentbasics2008Chapter5 propertyplantandequipmentbasics2008
Chapter5 propertyplantandequipmentbasics2008
 
Chapter4 inventory2008
Chapter4 inventory2008Chapter4 inventory2008
Chapter4 inventory2008
 
Chapter2 taxation2008
Chapter2 taxation2008Chapter2 taxation2008
Chapter2 taxation2008
 
Chapter1 the pillars_2008
Chapter1 the pillars_2008Chapter1 the pillars_2008
Chapter1 the pillars_2008
 

Kürzlich hochgeladen

Malegaon Call Girls Service ☎ ️82500–77686 ☎️ Enjoy 24/7 Escort Service
Malegaon Call Girls Service ☎ ️82500–77686 ☎️ Enjoy 24/7 Escort ServiceMalegaon Call Girls Service ☎ ️82500–77686 ☎️ Enjoy 24/7 Escort Service
Malegaon Call Girls Service ☎ ️82500–77686 ☎️ Enjoy 24/7 Escort ServiceDamini Dixit
 
Dr. Admir Softic_ presentation_Green Club_ENG.pdf
Dr. Admir Softic_ presentation_Green Club_ENG.pdfDr. Admir Softic_ presentation_Green Club_ENG.pdf
Dr. Admir Softic_ presentation_Green Club_ENG.pdfAdmir Softic
 
Marel Q1 2024 Investor Presentation from May 8, 2024
Marel Q1 2024 Investor Presentation from May 8, 2024Marel Q1 2024 Investor Presentation from May 8, 2024
Marel Q1 2024 Investor Presentation from May 8, 2024Marel
 
Organizational Transformation Lead with Culture
Organizational Transformation Lead with CultureOrganizational Transformation Lead with Culture
Organizational Transformation Lead with CultureSeta Wicaksana
 
Russian Call Girls In Rajiv Chowk Gurgaon ❤️8448577510 ⊹Best Escorts Service ...
Russian Call Girls In Rajiv Chowk Gurgaon ❤️8448577510 ⊹Best Escorts Service ...Russian Call Girls In Rajiv Chowk Gurgaon ❤️8448577510 ⊹Best Escorts Service ...
Russian Call Girls In Rajiv Chowk Gurgaon ❤️8448577510 ⊹Best Escorts Service ...lizamodels9
 
The Abortion pills for sale in Qatar@Doha [+27737758557] []Deira Dubai Kuwait
The Abortion pills for sale in Qatar@Doha [+27737758557] []Deira Dubai KuwaitThe Abortion pills for sale in Qatar@Doha [+27737758557] []Deira Dubai Kuwait
The Abortion pills for sale in Qatar@Doha [+27737758557] []Deira Dubai Kuwaitdaisycvs
 
Call Girls From Pari Chowk Greater Noida ❤️8448577510 ⊹Best Escorts Service I...
Call Girls From Pari Chowk Greater Noida ❤️8448577510 ⊹Best Escorts Service I...Call Girls From Pari Chowk Greater Noida ❤️8448577510 ⊹Best Escorts Service I...
Call Girls From Pari Chowk Greater Noida ❤️8448577510 ⊹Best Escorts Service I...lizamodels9
 
How to Get Started in Social Media for Art League City
How to Get Started in Social Media for Art League CityHow to Get Started in Social Media for Art League City
How to Get Started in Social Media for Art League CityEric T. Tung
 
Chandigarh Escorts Service 📞8868886958📞 Just📲 Call Nihal Chandigarh Call Girl...
Chandigarh Escorts Service 📞8868886958📞 Just📲 Call Nihal Chandigarh Call Girl...Chandigarh Escorts Service 📞8868886958📞 Just📲 Call Nihal Chandigarh Call Girl...
Chandigarh Escorts Service 📞8868886958📞 Just📲 Call Nihal Chandigarh Call Girl...Sheetaleventcompany
 
Phases of Negotiation .pptx
 Phases of Negotiation .pptx Phases of Negotiation .pptx
Phases of Negotiation .pptxnandhinijagan9867
 
Al Mizhar Dubai Escorts +971561403006 Escorts Service In Al Mizhar
Al Mizhar Dubai Escorts +971561403006 Escorts Service In Al MizharAl Mizhar Dubai Escorts +971561403006 Escorts Service In Al Mizhar
Al Mizhar Dubai Escorts +971561403006 Escorts Service In Al Mizharallensay1
 
Falcon Invoice Discounting: The best investment platform in india for investors
Falcon Invoice Discounting: The best investment platform in india for investorsFalcon Invoice Discounting: The best investment platform in india for investors
Falcon Invoice Discounting: The best investment platform in india for investorsFalcon Invoice Discounting
 
Uneak White's Personal Brand Exploration Presentation
Uneak White's Personal Brand Exploration PresentationUneak White's Personal Brand Exploration Presentation
Uneak White's Personal Brand Exploration Presentationuneakwhite
 
Lundin Gold - Q1 2024 Conference Call Presentation (Revised)
Lundin Gold - Q1 2024 Conference Call Presentation (Revised)Lundin Gold - Q1 2024 Conference Call Presentation (Revised)
Lundin Gold - Q1 2024 Conference Call Presentation (Revised)Adnet Communications
 
Call Girls Zirakpur👧 Book Now📱7837612180 📞👉Call Girl Service In Zirakpur No A...
Call Girls Zirakpur👧 Book Now📱7837612180 📞👉Call Girl Service In Zirakpur No A...Call Girls Zirakpur👧 Book Now📱7837612180 📞👉Call Girl Service In Zirakpur No A...
Call Girls Zirakpur👧 Book Now📱7837612180 📞👉Call Girl Service In Zirakpur No A...Sheetaleventcompany
 
Falcon Invoice Discounting: Unlock Your Business Potential
Falcon Invoice Discounting: Unlock Your Business PotentialFalcon Invoice Discounting: Unlock Your Business Potential
Falcon Invoice Discounting: Unlock Your Business PotentialFalcon investment
 
PHX May 2024 Corporate Presentation Final
PHX May 2024 Corporate Presentation FinalPHX May 2024 Corporate Presentation Final
PHX May 2024 Corporate Presentation FinalPanhandleOilandGas
 
Falcon's Invoice Discounting: Your Path to Prosperity
Falcon's Invoice Discounting: Your Path to ProsperityFalcon's Invoice Discounting: Your Path to Prosperity
Falcon's Invoice Discounting: Your Path to Prosperityhemanthkumar470700
 

Kürzlich hochgeladen (20)

Malegaon Call Girls Service ☎ ️82500–77686 ☎️ Enjoy 24/7 Escort Service
Malegaon Call Girls Service ☎ ️82500–77686 ☎️ Enjoy 24/7 Escort ServiceMalegaon Call Girls Service ☎ ️82500–77686 ☎️ Enjoy 24/7 Escort Service
Malegaon Call Girls Service ☎ ️82500–77686 ☎️ Enjoy 24/7 Escort Service
 
(Anamika) VIP Call Girls Napur Call Now 8617697112 Napur Escorts 24x7
(Anamika) VIP Call Girls Napur Call Now 8617697112 Napur Escorts 24x7(Anamika) VIP Call Girls Napur Call Now 8617697112 Napur Escorts 24x7
(Anamika) VIP Call Girls Napur Call Now 8617697112 Napur Escorts 24x7
 
Dr. Admir Softic_ presentation_Green Club_ENG.pdf
Dr. Admir Softic_ presentation_Green Club_ENG.pdfDr. Admir Softic_ presentation_Green Club_ENG.pdf
Dr. Admir Softic_ presentation_Green Club_ENG.pdf
 
unwanted pregnancy Kit [+918133066128] Abortion Pills IN Dubai UAE Abudhabi
unwanted pregnancy Kit [+918133066128] Abortion Pills IN Dubai UAE Abudhabiunwanted pregnancy Kit [+918133066128] Abortion Pills IN Dubai UAE Abudhabi
unwanted pregnancy Kit [+918133066128] Abortion Pills IN Dubai UAE Abudhabi
 
Marel Q1 2024 Investor Presentation from May 8, 2024
Marel Q1 2024 Investor Presentation from May 8, 2024Marel Q1 2024 Investor Presentation from May 8, 2024
Marel Q1 2024 Investor Presentation from May 8, 2024
 
Organizational Transformation Lead with Culture
Organizational Transformation Lead with CultureOrganizational Transformation Lead with Culture
Organizational Transformation Lead with Culture
 
Russian Call Girls In Rajiv Chowk Gurgaon ❤️8448577510 ⊹Best Escorts Service ...
Russian Call Girls In Rajiv Chowk Gurgaon ❤️8448577510 ⊹Best Escorts Service ...Russian Call Girls In Rajiv Chowk Gurgaon ❤️8448577510 ⊹Best Escorts Service ...
Russian Call Girls In Rajiv Chowk Gurgaon ❤️8448577510 ⊹Best Escorts Service ...
 
The Abortion pills for sale in Qatar@Doha [+27737758557] []Deira Dubai Kuwait
The Abortion pills for sale in Qatar@Doha [+27737758557] []Deira Dubai KuwaitThe Abortion pills for sale in Qatar@Doha [+27737758557] []Deira Dubai Kuwait
The Abortion pills for sale in Qatar@Doha [+27737758557] []Deira Dubai Kuwait
 
Call Girls From Pari Chowk Greater Noida ❤️8448577510 ⊹Best Escorts Service I...
Call Girls From Pari Chowk Greater Noida ❤️8448577510 ⊹Best Escorts Service I...Call Girls From Pari Chowk Greater Noida ❤️8448577510 ⊹Best Escorts Service I...
Call Girls From Pari Chowk Greater Noida ❤️8448577510 ⊹Best Escorts Service I...
 
How to Get Started in Social Media for Art League City
How to Get Started in Social Media for Art League CityHow to Get Started in Social Media for Art League City
How to Get Started in Social Media for Art League City
 
Chandigarh Escorts Service 📞8868886958📞 Just📲 Call Nihal Chandigarh Call Girl...
Chandigarh Escorts Service 📞8868886958📞 Just📲 Call Nihal Chandigarh Call Girl...Chandigarh Escorts Service 📞8868886958📞 Just📲 Call Nihal Chandigarh Call Girl...
Chandigarh Escorts Service 📞8868886958📞 Just📲 Call Nihal Chandigarh Call Girl...
 
Phases of Negotiation .pptx
 Phases of Negotiation .pptx Phases of Negotiation .pptx
Phases of Negotiation .pptx
 
Al Mizhar Dubai Escorts +971561403006 Escorts Service In Al Mizhar
Al Mizhar Dubai Escorts +971561403006 Escorts Service In Al MizharAl Mizhar Dubai Escorts +971561403006 Escorts Service In Al Mizhar
Al Mizhar Dubai Escorts +971561403006 Escorts Service In Al Mizhar
 
Falcon Invoice Discounting: The best investment platform in india for investors
Falcon Invoice Discounting: The best investment platform in india for investorsFalcon Invoice Discounting: The best investment platform in india for investors
Falcon Invoice Discounting: The best investment platform in india for investors
 
Uneak White's Personal Brand Exploration Presentation
Uneak White's Personal Brand Exploration PresentationUneak White's Personal Brand Exploration Presentation
Uneak White's Personal Brand Exploration Presentation
 
Lundin Gold - Q1 2024 Conference Call Presentation (Revised)
Lundin Gold - Q1 2024 Conference Call Presentation (Revised)Lundin Gold - Q1 2024 Conference Call Presentation (Revised)
Lundin Gold - Q1 2024 Conference Call Presentation (Revised)
 
Call Girls Zirakpur👧 Book Now📱7837612180 📞👉Call Girl Service In Zirakpur No A...
Call Girls Zirakpur👧 Book Now📱7837612180 📞👉Call Girl Service In Zirakpur No A...Call Girls Zirakpur👧 Book Now📱7837612180 📞👉Call Girl Service In Zirakpur No A...
Call Girls Zirakpur👧 Book Now📱7837612180 📞👉Call Girl Service In Zirakpur No A...
 
Falcon Invoice Discounting: Unlock Your Business Potential
Falcon Invoice Discounting: Unlock Your Business PotentialFalcon Invoice Discounting: Unlock Your Business Potential
Falcon Invoice Discounting: Unlock Your Business Potential
 
PHX May 2024 Corporate Presentation Final
PHX May 2024 Corporate Presentation FinalPHX May 2024 Corporate Presentation Final
PHX May 2024 Corporate Presentation Final
 
Falcon's Invoice Discounting: Your Path to Prosperity
Falcon's Invoice Discounting: Your Path to ProsperityFalcon's Invoice Discounting: Your Path to Prosperity
Falcon's Invoice Discounting: Your Path to Prosperity
 

Chapter9 noncurrentassetsheldforsale2008

  • 1. Gripping IFRS Non-current assets held for sale and discontinued operations Chapter 9 Non-current Assets Held for Sale and Discontinued Operations Reference: IFRS 5 Contents: Page 1. Introduction 310 2. Definitions 310 3. Non-current assets held for sale: identification 311 3.1 Overview 311 3.2 Criteria to be met before a non-current asset is classified as ‘held for sale’ 311 3.2.1 General criteria 311 3.2.2 Criteria where a completed sale is not expected within one year 311 3.2.3 Criteria where the asset is acquired with the intention to sell 312 4. Non-current assets held for sale: measurement 312 4.1 General measurement principles 312 4.2 Measurement principles specific to the cost model 313 4.2.1 The basic principles when the cost model was used 313 Example 1: reclassification of an asset measured using the cost model 313 Example 2: reclassification of an asset measured using the cost model 315 4.2.2 The tax effect when the cost model was used 316 Example 3: tax effect of reclassification and the cost model 316 4.3 Measurement principles specific to the revaluation model 317 4.3.1 The principles when the revaluation model was used 317 Example 4: reclassification of an asset using the revaluation model 318 Example 5: re-measurement of an asset held for sale, using the 320 revaluation model 4.4 Reversal of classification as ‘held for sale’ 321 Example 6: re-measurement of assets no longer ‘held for sale’ 322 5. Non-current assets held for sale: disclosure 322 5.1 Overview 322 5.2 In the statement of financial position 322 5.3 In the statement of financial position or notes thereto 323 5.4 Other note disclosure 323 Example 7: disclosure of non-current assets held for sale 323 6. Discontinued operations: identification 325 7. Discontinued operations: measurement 325 8. Discontinued operations: disclosure 326 8.1 In the statement of comprehensive income 326 8.2 In the statement of cash flows 327 8.3 Other note disclosure 328 8.3.1 Components no longer held for sale 328 8.3.2 Criteria met after the end of the reporting period 328 9. Summary 329 309 Chapter 9
  • 2. Gripping IFRS Non-current assets held for sale and discontinued operations 1. Introduction As its name suggests, this IFRS covers two areas, namely: • non-current assets held for sale; and • discontinued operations. With regard to ‘non-current assets’, this IFRS essentially suggests that there needs to be a further classification in the statement of financial position: ‘non-current assets held for sale’. In addition, it specifies that ‘held for sale assets’ are not to be depreciated. This IFRS does not apply to the following assets since these assets are covered by their own specific standards: • Deferred tax assets (IAS 12) • Assets relating to employee benefits (IAS 19) • Financial assets (IAS 39) • Investment property measured under the fair value model (IAS 40) • Non-current assets measured at fair value less point-of-sale costs (IAS 41: Agriculture) • Contractual rights under insurance contracts (IFRS 4) 2. Definitions Definitions included in Appendix A of the IFRS include the following: • Current asset: an asset - that is expected to be realised within 12 months after the end of the reporting period; - that is expected to be sold, used or realised (converted into cash) as part of the normal operating cycle; - that is held mainly for the purpose of being traded; or - that is a cash or cash equivalents that is not restricted in use within the 12 month period after the end of the reporting period. • non-current asset: an asset that does not meet the definition of a current asset • discontinued operation: a component of an entity that either has been disposed of or is classified as held for sale and: a) represents a separate major line of business or geographical area of operations, b) is part of a single co-ordinated plan to dispose of a separate major line of business or geographical area of operations; or c) is a subsidiary acquired exclusively with view to resale. • component of an entity: operations and cash flows that can be clearly distinguished, operationally and for financial reporting purposes, from the rest of the entity. • disposal group: a group of assets to be disposed of, by sale or otherwise, together as a group in a single transaction, and liabilities directly associated with those assets that will be transferred in the transaction. The group includes goodwill acquired in a business combination if the group is a cash-generating unit to which goodwill has been allocated in accordance with the requirements of paragraphs 80-87 of IAS 36 Impairment of Assets (as revised in 2004) or if it is an operation within a cash-generating unit. • firm purchase commitment: an agreement with an unrelated party, binding on both parties and usually legally enforceable, that: a) specifies all significant terms, including the price and timing of the transactions; and b) includes a disincentive for non-performance that is sufficiently large to make performance highly probable. • highly probable: significantly more likely than probable. • probable: more likely than not. 310 Chapter 9
  • 3. Gripping IFRS Non-current assets held for sale and discontinued operations 3. Non-current assets held for sale: identification (IFRS 5.6 - .12) 3.1 Overview The main thrust of IFRS 5 is that non-current assets that are ‘held for sale’ must be classified separately in the statement of financial position (i.e. a machine that is held for sale will no longer be included as part of property, plant and equipment). Certain criteria must first be met before a non-current asset is classified as a ‘non-current asset held for sale’. 3.2 Criteria to be met before a non-current asset is classified as ‘held for sale’ 3.2.1 General criteria A non-current asset (or disposal group) must be classified as held for sale if its carrying amount will be recovered mainly through a sale transaction than through continuing use. Non-current assets that meet all the following criteria may be separately classified as ‘non- current assets held for sale’: • Is the asset available for sale immediately and at normal terms? The asset (or disposal group) must be available for immediate sale in its present condition subject only to terms that are usual and customary for sales of such assets (or disposal groups); • Has management committed itself to a sales plan? Management, with the necessary authority to approve the action, must have committed itself to a plan to sell; • Has an active programme to sell begun? The active programme must be to both locate a buyer and to complete the plan to sell the asset (or disposal group); • Is the sale expected to happen within one year? The sale must be expected to qualify for recognition as a completed sale within one year from the date of classification as held for sale, except as permitted by paragraph 9 and appendix B; • Is the expected selling price reasonable? The asset (or disposal group) must be actively marketed at a price that is reasonable in relation to its current fair value; and • Is it unlikely that significant changes to the plan will be made? The actions required to complete the plan must indicate that it is unlikely that significant changes to the plan will be made or that the plan will be withdrawn. This means that assets that are to be abandoned should not be classified and measured as ‘held for sale’ since their carrying amount will be recovered principally through continuing use (until date of abandonment) rather than through a sale. This means that depreciation on assets that are to be abandoned should not cease. 3.2.2 Criteria where a completed sale is not expected within one year (Appendix B) There may be occasions where the asset would still be ‘held for sale’ even though the sale may not be completed and recognised as a sale within one year. This happens when: • At the date that the entity commits itself to a plan to sell a non-current asset (or disposal group), it reasonably expects that others (not a buyer) will impose conditions on the transfer of the asset (or disposal group) that will extend the period required to complete the sale, and: - actions necessary to respond to those conditions cannot be initiated until after a firm purchase commitment is obtained, and - a firm purchase commitment is highly probable within one year. • An entity obtains a firm purchase commitment and, as a result, a buyer or others unexpectedly impose conditions on the transfer of a non-current asset (or disposal group) previously classified as held for sale that will extend the period required to complete the sale, and: - timely actions necessary to respond to the conditions have been taken, and - a favourable resolution of the delaying factors is expected. 311 Chapter 9
  • 4. Gripping IFRS Non-current assets held for sale and discontinued operations • During the initial one-year period, circumstances arise that were previously considered unlikely and, as a result, a non-current asset (or disposal group) previously classified as held for sale is not sold by the end of the period, and: - during the initial one-year period the entity took action necessary to respond to the change in circumstances, - the non-current asset (or disposal group) is being actively marketed at a price that is reasonable, given the change in circumstances, and - the criteria in paragraph 7 (that sets out that the asset must be available for immediate sale) and paragraph 8 (that sets out that the sale must be highly probable) are met. 3.2.3 Criteria where the asset is acquired with the intention to sell (IFRS 5.11) It may happen that an entity acquires a non-current asset (or disposal group) exclusively with the view to its subsequent disposal. In this case, the non-current asset must be classified as ‘held for sale’ immediately on acquisition date, on condition that: • the one-year requirement is met (unless a longer period is allowed by paragraph 9 and the related appendix B); and • it is highly probable that any other criteria given in para 7 and para 8 that are not met immediately on the date of acquisition, will be met within a short period (usually three months) after acquisition. 4. Non-current assets held for sale: measurement (IFRS 5.15 - .25) 4.1 General measurement principles An entity shall measure a non-current asset (or disposal group) classified as held for sale at the lower of its carrying amount and fair value less costs to sell. If a newly acquired asset (or disposal group) meets the criteria to be classified as held for sale, applying paragraph 15 will result in the asset being measured on initial recognition at the lower of its carrying amount had it not been so classified (e.g. cost) and fair value less costs to sell. Since the asset is newly acquired, its cost will equal its fair value. Therefore, an asset acquired as part of a business combination, shall initially be measured at fair value (its cost) less costs to sell. For all other assets (other than newly acquired assets) that are classified as non-current assets held for sale, there are two distinct phases of its life: • Before it was classified as held for sale; and • Once it is classified as held for sale. Before an asset is classified as held for sale, it is measured in terms of its own relevant IFRS. If, for example, the asset is an item of property, plant and equipment, the asset will have been measured in terms of IAS 16, which will mean that: • on initial acquisition, the asset will have been recorded at cost; and • subsequently, the asset will have been depreciated, revalued (if the revaluation model was used to measure the asset) and reviewed for impairments annually (whether the cost or revaluation model were used). If this asset is then to be reclassified as ‘held for sale’, it will be measured as follows: • In terms of its previous relevant IFRS: Immediately before reclassifying the asset as ‘held for sale’, the asset must be re- measured using its previous measurement model; for example if the asset was previously an item of property, plant and equipment that was measured using the: • Cost model: depreciate to date of reclassification and then check for impairments; or • Revaluation model: depreciate to date of reclassification, revalue if appropriate and check for impairments; then 312 Chapter 9
  • 5. Gripping IFRS Non-current assets held for sale and discontinued operations • In terms of IFRS 5: On reclassifying the asset as ‘held for sale’, - re-measure to the lower of ‘carrying amount’ and ‘fair value less costs to sell’; and - stop depreciating it. If, in the unusual instance a sale is not expected to occur within one year, it may be necessary (depending on materiality) to measure the ‘costs to sell’ at their present value. 4.2 Measurement principles specific to the cost model 4.2.1 The basic principles when the cost model was used If an asset measured under the cost model is re-classified as ‘held for sale’: • immediately before reclassifying the asset as ‘held for sale’, the asset must be re- measured using its previous measurement model (i.e. the cost model per IAS 16, if the item was previously property, plant and equipment); • then, in terms of IFRS 5: - re-measure it to the lower of ‘carrying amount’ and ‘fair value less costs to sell’; - stop depreciating it; and - re-measure to ‘fair value less costs to sell’ whenever appropriate: any impairment loss will be expensed in the statement of comprehensive income whereas impairment losses reversed are recognised as income but are limited to the asset’s accumulated impairment losses. You may have noticed that, when using the cost model, there can be no initial increase in the carrying amount on classification as ‘held for sale’ because the non-current asset must initially be measured at the lower of its ‘carrying amount’ and ‘fair value less costs to sell’. For example, an asset with a ‘carrying amount’ of 80 000 and ‘fair value less costs to sell’ of 90 000 will not be adjusted because the lower of the two is the current carrying amount of 80 000. Example 1: reclassification of an asset measured using the cost model An item of plant, measured using the cost model, has a carrying amount of C80 000 (cost: 100 000 and accumulated depreciation: 20 000) on 1 January 20X3 on which date all criteria for separate classification as a ‘non-current asset held for sale’ are met. Required: Show the journal entries relating to the reclassification of the plant assuming that: A. the fair value is C70 000 and the expected costs to sell are C5 000 on 1 January 20X3; B. on 30 June 20X3 (6-months later), the fair value is C70 000 and expected costs to sell are C2 000; C. on 30 June 20X3 (6-months later), the fair value is C90 000 and expected costs to sell are C5 000. Solution to example 1: reclassification of an asset using the cost model Comment: this example explains the limit to the reversal of the impairment loss. A. If carrying amount > ‘fair value less costs to sell’: recognise an ‘impairment loss’ (expense) Workings: C Carrying amount given 80 000 Fair value less costs to sell: 70 000 – 5 000 (65 000) Decrease in value (impairment loss) 80 000 – 65 000 15 000 313 Chapter 9
  • 6. Gripping IFRS Non-current assets held for sale and discontinued operations Journal: 1 January 20X3 Debit Credit Impairment loss (expense) 15 000 - Plant: accumulated impairment loss 15 000 Impairment loss before initial classification as ‘held for sale’ Note: There is no depreciation on this asset. B. If ‘fair value less costs to sell’ subsequently increases: recognise a ‘reversal of impairment loss’ (income) – limited to accumulated impairment losses Workings: C New fair value less costs to sell: 70 000 – 2 000 68 000 Prior fair value less costs to sell: 100 000 cost – 20 000 accum depreciation – (65 000) 15 000 impairment loss Impairment loss reversed*: 68 000 – 65 000 3 000 * Note: the ‘accumulated impairment loss’ is 15 000 before the reversal, thus the reversal of 3 000 is not limited (the previous accumulated impairment loss is bigger: 15 000 is bigger than 3 000). Journal: 30 June 20X3 Debit Credit Plant: accumulated impairment loss 3 000 - Impairment loss reversed (income) 3 000 Reversal of impairment loss: on re-measurement of ‘NCA held for sale’ Note: There is no depreciation on this asset. The impairment to date is C12 000 (15 000 – 3 000) C. If ‘fair value less costs to sell’ subsequently increases: recognise a ‘reversal of impairment loss’ (income) – limited to accumulated impairment losses Workings: C New fair value less costs to sell: 90 000 – 5 000 85 000 Prior fair value less costs to sell 100 000 – 20 000 accum depreciation – (65 000) 15 000 impairment loss Increase in value 20 000 Limited to prior cumulative impairment losses 15 000 Impairment loss reversed*: 85 000 – 65 000 = 20 000 limited to 15 000 15 000 * Note: the difference between the latest ‘fair value less costs to sell’ (85 000) and the prior ‘fair value less costs to sell’ (65 000) of 20 000 is limited to the previous ‘accumulated impairment loss’ of 15 000. Journal: 30 June 20X3 Debit Credit Plant: accumulated impairment loss 15 000 - Impairment loss reversed (income) 15 000 Reversal of impairment loss on re-measurement of ‘non-current asset held for sale’ Note: There is no depreciation on this asset. The impairment to date is C0 (15 000 – 15 000) 314 Chapter 9
  • 7. Gripping IFRS Non-current assets held for sale and discontinued operations Example 2: reclassification of an asset measured using the cost model An item of plant, measured using the cost model (i.e. at historical carrying amount), has a carrying amount of 80 000 (cost 100 000) on 1 January 20X3 on which date all criteria for separate classification as a ‘non-current asset held for sale’ are met. This asset had previously been impaired by 3 000 (i.e. this is the balance on the accumulated impairment loss account). Required: Show the journal entries relating to the reclassification of the plant assuming: A. the fair value is 70 000 and the expected costs to sell are 5 000 on 1 January 20X3; B. 6 months later, on 30 June 20X3, the fair value is 70 000 and the expected costs to sell are 2 000; C. 6 months later, on 30 June 20X3, the fair value is 90 000 and the expected costs to sell are 5 000. Solution to example 2: reclassification of an asset measured using the cost model Comment: this example explains the limit to the reversal of the impairment loss. It differs from the previous example in that this asset had previously been impaired before it was reclassified as a non- current asset held for sale. A. If carrying amount > ‘fair value less costs to sell’: recognise an ‘impairment loss’ (expense) Workings: C Carrying amount given 80 000 Fair value less costs to sell: 70 000 – 5 000 (65 000) Decrease in value (impairment loss) 80 000 – 65 000 15 000 Journal: 1 January 20X3 Debit Credit Impairment loss (expense) 15 000 - Plant: accumulated impairment loss 15 000 Impairment loss on initial classification of NCA as ‘held for sale’ Note: There is no depreciation on this asset. The impairment to date is now C18 000 (3 000 + 15 000) B. If ‘fair value less costs to sell’ subsequently increases: recognise a ‘reversal of impairment loss’ (income) – limited to accumulated impairment losses Workings: C New fair value less costs to sell 70 000 – 2 000 68 000 Prior fair value less costs to sell 70 000 – 5 000 (65 000) Increase in value (impairment loss reversed*) 68 000 – 65 000 3 000 * Note: the ‘accumulated impairment loss’ is 18 000 before this reversal (15 000 + 3 000), therefore the impairment loss reversal of 3 000 is not limited (the previous accumulated impairment loss is bigger: 18 000 is bigger than 3 000). Journal: 30 June 20X3 Debit Credit Plant: accumulated impairment loss 3 000 - Impairment loss reversed (income) 3 000 Reversal of impairment loss on re-measurement of ‘asset held for sale’ Note: There is no depreciation on this asset. The impairment to date is now C15 000 (18 000 - 3 000) 315 Chapter 9
  • 8. Gripping IFRS Non-current assets held for sale and discontinued operations C. If ‘fair value less costs to sell’ subsequently increases: recognise a ‘reversal of impairment loss’ (income) – limited to accumulated impairment losses Workings: C New fair value less costs to sell: 90 000 – 5 000 85 000 Prior fair value less costs to sell 70 000 – 5 000 (65 000) Increase in value 20 000 Limited to prior cumulative impairment losses 15 000 + 3 000 18 000 Impairment loss reversed*: 85 000 – 65 000 = 20 000 limited to 15 000 18 000 * Note: The difference between the latest ‘fair value less costs to sell’ and the prior ‘fair value less costs to sell’ of 20 000 is limited to the ‘cumulative impairment loss’ recognised of 18 000, calculated as follows: C Impairment loss: 18 000 - before reclassification given 3 000 - on reclassification 80 000 – 65 000 15 000 Journal: 30 June 20X3 Debit Credit Plant: accumulated impairment loss 18 000 - Impairment loss reversed (income) 18 000 Reversal of impairment loss on re-measurement of ‘asset held for sale’ Note: There is no depreciation on this asset. The impairment to date is now C0 (18 000 - 18 000) 4.2.2 The tax effect when the cost model was used As soon as an asset is classified as held for sale, depreciation thereon ceases. The tax authorities, however, do not stop deducting tax allowances (where tax allowances were due in terms of the tax legislation) simply because you have decided to sell the asset. The difference between the nil depreciation and the tax allowance (if appropriate) causes deferred tax. The principles affecting the current tax payable and deferred tax balances are therefore exactly the same as for any other non-current asset. Example 3: tax effect of reclassification and the cost model An item of plant, measured using the cost model (i.e. at historical carrying amount), has a carrying amount of C70 000 (cost 100 000) and a tax base of C90 000 on 1 January 20X3 on which date all criteria for separate classification as a ‘non-current asset held for sale’ are met. The fair value less costs to sell on this date are C65 000. This asset had not previously been impaired. The tax authorities allow a deduction of 10% on the cost of this asset. The tax rate is 30%. The profit before tax is correctly calculated to be C200 000. There are no temporary or permanent differences other than those evident from the information provided. Required: A. Calculate the current normal tax payable and the deferred tax balance at 31 December 20X3. B. Journalise the current normal tax and the deferred tax for the year ended 31 December 20X3. 316 Chapter 9
  • 9. Gripping IFRS Non-current assets held for sale and discontinued operations Solution to example 3: tax effect of reclassification and the cost model A: Calculations Current normal income tax Calculations C Profit before tax 200 000 Add back depreciation Assets held for sale are not depreciated 0 Add back impairment Impairment on re-classification as ‘held for sale’ 5 000 Less tax allowance 100 000 x 10% (10 000) Taxable profits 195 000 Current tax 195 000 x 30% 58 500 Deferred tax: Carrying Tax Temporary Deferred Non-current asset held for sale amount base difference tax Balance – 1 January 20X3 70 000 90 000 20 000 6 000 Asset Less impairment to ‘fair value – (5 000) 0 Cr DT, costs to sell’ (70 000 – 65 000) (1 500) Dr TE Depreciation/ tax allowance 0 (10 000) Balance – 31 December 20X3 65 000 80 000 15 000 4 500 Asset B: Journals 31 December 20X3 Debit Credit Tax expense 58 500 Current tax payable (liability) 58 500 Current normal tax payable (estimated) Tax expense 1 500 Deferred tax (liability) 1 500 Deferred tax adjustment 4.3 Measurement principles specific to the revaluation model 4.3.1 The principles when the revaluation model was used If an asset measured under the revaluation model is reclassified as ‘held for sale’: • immediately before reclassifying the asset as ‘held for sale’, the asset must be re- measured using its previous measurement model (i.e. the revaluation model per IAS 16); • then, in terms of IFRS 5: - re-measure it to the lower of ‘carrying amount’ and ‘fair value less costs to sell’; - stop depreciating it; and - then re-measure it to ‘fair value less costs to sell’ whenever appropriate: any further impairment loss (e.g. the selling costs) is expensed (even if there is a revaluation surplus) whereas an impairment loss reversed is recognised as income but is limited to the asset’s accumulated impairment losses. 317 Chapter 9
  • 10. Gripping IFRS Non-current assets held for sale and discontinued operations Example 4: reclassification of an asset measured using the revaluation model An item of plant, revalued to fair value using the revaluation model, met all criteria for classification as ‘held for sale’ on 1 January 20X4. The following information is relevant: Cost: 100 000 (purchased 1 January 20X1) Depreciation: 10% per annum straight-line to nil residual values. Fair value: 120 000 (revalued 1 January 20X3). Revaluations are performed using the net replacement value method Required: Show all journal entries relating to the reclassification as ‘held for sale’ assuming that: A. The fair value is C100 000 and the expected selling costs are C9 000 on 1 January 20X4; B. The fair value is C150 000 and the expected selling costs are C20 000 on 1 January 20X4. C. The fair value is C60 000 and the expected selling costs are C20 000 on 1 January 20X4. Solution to example 4: reclassification of an asset measured using the revaluation model A. If the actual carrying amount > historical carrying amount (i.e. there is already a revaluation surplus) and the fair value decreases on date of reclassification (although not entirely removing the revaluation surplus balance) and there are costs to sell: reverse revaluation surplus due to drop in fair value and recognise selling costs as an ‘impairment loss’ (expense) Workings: C Fair value (1 January 20X3) 120 000 Accumulated depreciation (31 December 20X3: since 120 000/ 8 remaining years (15 000) the revaluation on 1 January 20X3) Actual carrying amount (1 January 20X4): 120 000 – 15 000 105 000 Fair value Given (100 000) Decrease in value (all through revaluation surplus) See below for calculation of RS balance 5 000 Actual carrying amount (1 January 20X4): 120 000 – 15 000 (above) 105 000 Historical carrying amount (1 January 20X4) 100 000/ 10 years x 7 years (70 000) Balance on the revaluation surplus (1 January 20X4): Proof: (120 000 – 80 000) / 8 x 7 years 35 000 Decrease in value (above) (5 000) Balance on the revaluation surplus (1 January 20X4): Further balance against which further 30 000 devaluation would be processed (IAS16) Journals: 1 January 20X4 Debit Credit Plant: accumulated depreciation and impairment losses 15 000 - Plant: cost 15 000 NRVM: Accumulated depreciation set-off against cost Revaluation surplus FV: C100 000 – Carrying amount: C105 000 5 000 - Plant: cost 5 000 Re-measurement to FV before reclassification Impairment loss (selling costs) (expense) 9 000 - Plant: accumulated depreciation and impairment losses 9 000 Re-measurement to lower of CA or FV less costs to sell on reclassification: CA: 100 000 – FV less Costs to Sell: (100 000 – 9 000) Note: There is no further depreciation on this asset. 318 Chapter 9
  • 11. Gripping IFRS Non-current assets held for sale and discontinued operations B. If the actual carrying amount > historical carrying amount (i.e. there is already a revaluation surplus) and fair value increases and there are expected costs to sell: increase revaluation surplus due to increase in fair value and recognise the expected selling costs as an ‘impairment loss’ (expense) Workings: C Fair value 120 000 Accumulated depreciation (31 December 20X3: 120 000/ 8 remaining years (15 000) since the revaluation on 1 January 20X3) Actual carrying amount (1 January 20X4): 120 000 – 15 000 105 000 Fair value given 150 000 Increase in value (all through revaluation surplus) Through revaluation surplus (45 000) because carrying amount is already above the HCA: 100 000 / 10 x 7 Journals: 1 January 20X4 Debit Credit Plant: accumulated depreciation and impairment losses 15 000 - Plant: cost 15 000 NRVM: Accumulated depreciation set-off against cost: 120 000/ 8 years remaining on date of revaluation Plant: cost 45 000 - Revaluation surplus 45 000 Re-measurement to FV before reclassification: FV: 150 000 – Carrying amount: 105 000 Impairment loss (selling costs) (expense) 20 000 - Plant: accumulated depreciation and impairment losses 20 000 Re-measurement to lower of CA or FV less costs to sell on reclassification: Carrying amount: 150 000 – FV less costs to sell: (150 000 – 20 000) Note: There is no further depreciation on this asset. C. If the actual carrying amount > historical carrying amount (i.e. there is already a revaluation surplus) and fair value decreases removing the entire balance on the revaluation surplus and there are expected costs to sell: reverse revaluation surplus due to decrease in fair value and recognise the expected selling costs as an ‘impairment loss’ (expense) Workings: C Fair value 120 000 Accumulated depreciation (31 December 20X3: since 120 000/ 8 years (15 000) the revaluation on 1 January 20X3) Actual carrying amount (1 January 20X4): 120 000 – 15 000 105 000 Fair value given (60 000) Decrease in value (all through revaluation surplus) See below for calculation of RS bal 45 000 Actual carrying amount (1 January 20X4): 120 000 – 15 000 105 000 Historical carrying amount (1 January 20X4) 100 000/ 10years x 7 years (70 000) Balance on the revaluation surplus (1 January 20X4): (120 000 – 80 000) / 8 x 7 years 35 000 Decrease in value (above) 45 000 Reversal: revaluation surplus balance Balance in this account (above) 35 000 Impairment loss (balancing figure) 45 000 – 35 000 10 000 319 Chapter 9
  • 12. Gripping IFRS Non-current assets held for sale and discontinued operations Journals: 1 January 20X4 Debit Credit Plant: accumulated depreciation and impairment losses 15 000 - Plant: cost 15 000 NRVM: Accumulated depreciation set-off against cost: 120 000/ 8 years remaining on date of revaluation Revaluation surplus (ACA: 105 000 – HCA: 70 000) 35 000 Impairment loss (HCA: 70 000 – FV: 60 000) 10 000 - Plant: cost 35 000 - Plant: accumulated depreciation and impairment losses 10 000 Re-measurement to FV before reclassification: FV: 60 000 – CA: 105 000 Impairment loss (selling costs) (expense) 20 000 - Plant: accumulated depreciation and impairment losses 20 000 Re-measurement to lower of CA or FV less costs to sell on reclassification: CA: 60 000 – FV less costs to sell (60 000 – 20 000) Note: There is no further depreciation on this asset. Example 5: re-measurement of an asset held for sale using the revaluation model An item of plant, revalued to fair value using the revaluation model, met all criteria for classification as ‘held for sale’ on 1 January 20X4. The following information is relevant: Cost: 100 000 (purchased 1 January 20X1) Depreciation: 10% per annum straight-line to nil residual values. Fair value: 120 000 (revalued 1 January 20X3). Revaluations are performed using the net replacement value method The ‘fair value less costs to sell’ on 1 January 20X4 was as follows: • Fair value (1 January 20X4): 100 000; and • Expected selling costs (1 January 20X4): 9 000. Required: Show all journal entries relating to the re-measurement of the ‘non-current asset held for sale’ on 30 June 20X4 assuming that on the 30 June 20X4: A. The fair value is 110 000 and the expected selling costs are 15 000; B. The fair value is C110 000 and the expected selling costs are C3 000; C. The fair value is 90 000 and the expected selling costs are 3 000. Solution to example 5: re-measurement of an asset held for sale: the revaluation model Comment: this example explains the limit on the impairment loss that may be reversed. A. If the new fair value less costs to sell > previous fair value less costs to sell: reverse the impairment loss limited to prior cumulative impairment losses Workings: C New fair value less costs to sell (30 June 20X4) 110 000 (FV) – 15 000 (cost to sell) 95 000 Prior fair value less costs to sell (1 January 20X4) 100 000 (FV) – 9 000 (costs to sell) (91 000) Increase in value 4 000 Limited to prior cumulative impairment losses 100 000 (FV before reclassification) – 9 000 91 000 (FV – costs to sell) Therefore: impairment loss reversed Maximum that may be reversed is 9 000; 4 000 thus there is no limitation to the reversal in this case 320 Chapter 9
  • 13. Gripping IFRS Non-current assets held for sale and discontinued operations Journals: 30 June 20X4 Debit Credit Plant: accumulated depreciation and impairment losses 4 000 - Impairment loss reversed (income) 4 000 Re-measurement of non-current asset held for sale: increase in fair value less costs to sell B. If the new fair value less costs to sell > previous fair value less costs to sell: reverse the impairment loss limited to prior cumulative impairment losses Workings: C New fair value less costs to sell (30 June 20X4) 110 000 (FV) – 3 000 (cost to sell) 107 000 Prior fair value less costs to sell (1 January 20X4) 100 000 (FV) – 9 000 (costs to sell) (91 000) Increase in value 16 000 Limited to prior cumulative impairment losses 100 000 (FV before reclassification) – 9 000 91 000 (FV – costs to sell) Therefore: reversal of impairment loss 9 000 Journals: 30 June 20X4 Debit Credit Plant: accumulated impairment loss 9 000 - Reversal of impairment loss (income) 9 000 Re-measurement of non-current asset held for sale: increase in fair value less costs to sell (limited to 9 000) C. If the new fair value less costs to sell < previous fair value less costs to sell: recognise a further impairment loss Workings: C New fair value less costs to sell (30 June 20X4) 90 000 (FV) – 3 000 (cost to sell) 87 000 Prior fair value less costs to sell (1 January 20X4) 100 000 (FV) – 9 000 (costs to sell) 91 000 Decrease in value (impairment loss) 4 000 Journals: 30 June 20X4 Debit Credit Impairment loss (expense) 4 000 - Plant: accumulated depreciation and impairment losses 4 000 Re-measurement of non-current asset held for sale: decrease in fair value less costs to sell 4.4 Reversal of classification as ‘held for sale’ (IFRS 5.26 - .29) If a non-current asset that was previously classified as ‘held for sale’ no longer meets the criteria necessary for such a classification, the asset must immediately cease to be classified as ‘held for sale’ and must be re-measured to the lower of: • its carrying amount had the non-current asset never been classified as ‘held for sale’(adjusted for any depreciation, amortisation and/ or revaluations that would have been recognised had the asset not been classified as held for sale); and • its recoverable amount. 321 Chapter 9
  • 14. Gripping IFRS Non-current assets held for sale and discontinued operations Example 6: re-measurement of assets no longer classified as ‘held for sale’ Plant, with a cost of C100 000 (1 January 20X1) and accumulated depreciation of C20 000 on 31 December 20X2 (10% straight-line for 2 years), was reclassified as ‘held for sale’ on 31 December 20X2 and immediately impaired to its ‘fair value less costs to sell’ of C65 000. On 30 June 20X3 (six months later), it ceased to meet all criteria necessary for classification as ‘held for sale’. On this date its recoverable amount is determined to be C85 000. Required: Show all journal entries relating to the re-measurement of plant previously held as a ‘non- current asset held for sale’. Solution to example 6: re-measurement of assets no longer classified as ‘held for sale’ Workings: C New carrying amount (30 June 20X3) to be lower of: 75 000 • Carrying amount had the asset never 100 000 – 20 000 – 100 000 x 10% x 6/12 75 000 been classified as ‘held for sale’ • Recoverable amount Given 85 000 Current carrying amount (30 June 20X3) Fair value – costs to sell (65 000) Impairment loss to be reversed 10 000 Journals: Debit Credit 30 June 20X3 Plant: accumulated impairment loss 10 000 - Impairment loss reversed (income) 10 000 Reversal of impairment loss on reclassification of ‘non-current asset held for sale’ as ‘property, plant and equipment’: criteria no longer met Note: Depreciation on this asset will now begin again. 5. Non-current assets held for sale: disclosure (IFRS 5.30 and .38 - .42) 5.1 Overview Extra disclosure is required where the financial statements include either: • a ‘non-current asset held for sale’; or • a ‘sale of a non-current asset’. The classification affects the period during which it was classified as ‘held for sale’. This means that no adjustment should be made to the measurement or presentation of the affected assets in the comparative periods presented. 5.2 In the statement of financial position Non-current assets (or non-current assets within a disposal group) that are ‘held for sale’ must be shown separately in the statement of financial position. If a disposal group includes liabilities, these liabilities must also be shown separately from other liabilities in the statement of financial position and may not be set-off against the assets within the disposal group. 322 Chapter 9
  • 15. Gripping IFRS Non-current assets held for sale and discontinued operations 5.3 In the statement of financial position or notes thereto Major classifications of assets within the total of the ‘non-current assets held for sale’ and major classifications of liabilities within the total ‘liabilities of a disposal group’ must be shown in the notes (unless shown in the statement of financial position). 5.4 Other note disclosure An entity shall disclose the following information in the notes in the period in which a non- current asset (or disposal group) has been classified as held for sale or sold: a) a description of the non-current asset (or disposal group); b) a description of the facts and circumstances of the sale, or leading to the expected disposal, and the expected manner and timing of that disposal; c) the gain or loss recognised in accordance with IFRS 5 (paragraph 20-22) and, if not separately presented in the statement of comprehensive income, the caption in the statement of comprehensive income that includes that gain or loss; d) if applicable, the segment in which the non-current asset (or disposal group) is presented in accordance with IAS 14 Segment Reporting. If, during the current period, there was a decision to reverse the plan to sell the non-current asset (or disposal group), the following extra disclosure would be required: a) the description of the facts and circumstances leading to the decision not to sell; and b) the effect of the decision on the results of operations for all periods presented. Example 7: disclosure of non-current assets held for sale Assume that an entity owns only the following non-current assets: • Plant; and • Factory buildings. Details of the plant are as follows: • Plant was purchased on 1 January 20X1 at a cost of C100 000; • Depreciation is provided over 10 years to a nil residual value on the straight-line basis; • Plant was reclassified as ‘held for sale’ on 31 December 20X2 and immediately impaired to its ‘fair value less costs to sell’ of C65 000; • On 30 June 20X3 (six months later), plant ceased to meet all criteria necessary for classification as ‘held for sale’, on which date its recoverable amount is C85 000. Details of the factory buildings are as follows: • The factory buildings were purchased on 1 January 20X1 at a cost of C600 000, • Depreciation is provided over 10 years to nil residual values on the straight-line basis • Factory buildings were reclassified as ‘held for sale’ on 30 June 20X3’ at a ‘fair value less cost to sell’ of C445 000. Required: Disclose all information necessary in relation to the plant and factory buildings in the financial statements for the year ended 31 December 20X3. 323 Chapter 9
  • 16. Gripping IFRS Non-current assets held for sale and discontinued operations Solution to example 7: disclosure of non-current assets held for sale Comment: this example explains how to disclose non-current assets held for sale, as well as how to disclose a non-current asset that is no longer held for sale. Company name Statement of financial position At 31 December 20X3 20X3 20X2 Non-current assets C C Property, plant and equipment 26 70 000 480 000 Non-current assets (and disposal groups) held for sale 27 445 000 65 000 Non-current liabilities Liabilities of a disposal group (for disclosure purposes only) 27 xxx xxx Company name Notes to the financial statements For the year ended 31 December 20X3 20X3 20X2 C C 5. Profit before tax Profit before tax is stated after taking into consideration the following (income)/ expenses: Depreciation – factory building 30 000 60 000 Depreciation – plant 5 000 10 000 Impairment loss – asset held for sale 5 000 15 000 Impairment loss reversed – asset no longer held for sale (10 000) 0 26. Property, plant and equipment Factory building 0 480 000 Plant 70 000 0 70 000 480 000 Factory building: Net carrying amount – 1 January 480 000 540 000 Gross carrying amount – 1 January 600 000 600 000 Accumulated depreciation and impairment losses – 1 January (120 000) (60 000) Depreciation (to 30 June 20X5) (30 000) (60 000) Impairment loss (to fair value less costs to sell: 450 000 – 445 000) (5 000) 0 Non-current asset now classified as ‘held for sale’ (445 000) 0 Net carrying amount – 31 December 0 480 000 Gross carrying amount – 31 December 0 600 000 Accumulated depreciation and impairment losses – 31 December 0 (120 000) Plant: Net carrying amount – 1 January 0 90 000 Gross carrying amount – 1 January 0 100 000 Accumulated depreciation and impairment losses – 1 January 0 (10 000) Non-current asset no longer classified as ‘held for sale’ 65 000 0 Reversal of impairment loss (to lower of HCA: 75 000 or RA:85 000) 10 000 0 Depreciation (20X3: 75 000 / 7,5 remaining years x 6/12) (5 000) (10 000) Impairment loss (to fair value less costs to sell: 80 000 – 65 000) 0 (15 000) Non-current asset now classified as ‘held for sale’ 0 (65 000) Net carrying amount – 31 December 70 000 0 Gross carrying amount – 31 December 100 000 0 Accumulated depreciation and impairment losses – 31 December (30 000) 0 324 Chapter 9
  • 17. Gripping IFRS Non-current assets held for sale and discontinued operations 27. Non-current assets held for sale 20X3 20X2 C C Factory buildings 445 000 0 Plant 0 65 000 Less non-current interest bearing liabilities (disclosure purpose) 0 0 445 000 65 000 The company is transferring its business to a new location and thus the existing factory building is to be sold (circumstances leading to the decision). The sale is expected to take place within 7 months of the end of the reporting period (expected timing). The factory building is expected to be sold as a going concern (expected manner of sale). The plant is no longer classified as ‘held for sale’ since it is now intended to be redeployed to other existing factories rather than to be sold together with the factory buildings (reasons for the decision not to sell). The effect on current year profit from operations is as follows: C - Gross (Impairment loss reversed: 10 000 – deprec.:5 000) 5 000 - Tax (1 500) - Net 3 500 6. Discontinued operations: identification (IAS 5.31 - .36) IFRS 5 requires that, where a component is identified as a discontinued operation, it must be separately disclosed in the financial statements. The following definitions are provided in IFRS 5: A component of an entity comprise: • operations and cash flows • that can be clearly distinguished, operationally and for financial reporting purposes, • from the rest of the entity. A component of an entity may be a cash-generating unit or any group thereof. A discontinued operation is • a component of an entity that has either been - disposed of, or - is classified as held for sale; • and meets one of the following criteria: - represents a separate major line of business or geographical area of operations; or - is part of a single co-ordinated plan to dispose of a separate major line of business or geographical area of operations; or - is a subsidiary acquired exclusively with a view to resale. 7. Discontinued operations: measurement A discontinued operation is, in effect, constituted by non-current assets (or disposal groups) held for sale that, together, comprise a component that meets the definition of a ‘discontinued operation’. Therefore, the principles that are adopted when measuring the individual non- current assets (or disposal groups) held for sale are also used when measuring the elements of a discontinued operation. If the non-current asset (or disposal group) does not meet the definition of a ‘component’, the related transactions and adjustments will not be disclosed as ‘discontinued operations’ but rather as part of ‘continuing operations’. 325 Chapter 9
  • 18. Gripping IFRS Non-current assets held for sale and discontinued operations 8. Discontinued operations: disclosure 8.1 In the statement of comprehensive income A single amount must be presented on the face of the statement of comprehensive income being the total of: • the post-tax profit or loss of the discontinued operations; • the post-tax gain or loss recognised on measurement to fair value less costs to sell; and • the post-tax gain or loss recognised on disposal of assets/ disposal groups making up the discontinued operations. An analysis of this single amount that is presented in the statement of comprehensive income must be presented ‘for all periods presented’. This analysis may be done in the statement of comprehensive income (see suggested presentation option A on the next page) or in the notes (see suggested presentation option B on the next page) and must show the following: • revenue of discontinued operations; • expenses of discontinued operations; • profit (or loss) before tax of discontinued operations; and • tax expense of discontinued operations. An entity must also disclose the following either in the statement of comprehensive income or in the notes thereto ‘for all periods presented’ (with the exception of the change in estimate): • gain or loss on re-measurement to ‘fair value less selling costs’; • gain or loss on disposal of the discontinued operation (made up by assets/ disposal groups); • tax effects of the above; and • changes to estimates made in respect of discontinued operations disposed of in a prior period (showing nature and amount); examples of such changes include outcomes of previous uncertainties relating to: - the disposal transaction (e.g. adjustments to the selling price); and - the operations of the component before its disposal (e.g. adjustments to warranty/ legal obligations retained by the entity). Option A: If the analysis of the profit or loss is presented on the face of the statement of comprehensive income, the statement of comprehensive income will look something like this (the figures are all assumed): Example Ltd Statement of comprehensive income For the year ended 31 December 20X3 (extracts) 20X3 20X3 20X3 20X2 20X2 20X2 C’000 C’000 C’000 C’000 C’000 C’000 Continuing Discontinued Total Continuing Discontinued Total Revenue 800 150 800 790 Expenses (300) (100) (400) (500) Profit before tax 500 50 400 290 Taxation expense (150) (60) (180) (97) Gains/ (losses) after tax 40 7 Gain/ (loss): re-measurement 30 10 to fair value less costs to sell Gain/ (loss): disposal of assets 20 0 in the discontinued operations Tax on gains/ (losses) (10) (3) Profit for the period 350 30 380 220 200 420 Other comprehensive income 0 0 0 0 0 0 Total comprehensive income 350 30 380 220 200 420 326 Chapter 9
  • 19. Gripping IFRS Non-current assets held for sale and discontinued operations Option B: If the total profit or loss is presented in the statement of comprehensive income, with the analysis in the notes, the statement of comprehensive income and notes will look something like this (the figures are all assumed): Example Ltd Statement of comprehensive income For the year ended 31 December 20X3 (extracts) 20X3 20X2 Note C’000 C’000 Revenue 800 800 Expenses (300) (400) Profit before tax 500 400 Taxation expense (150) (180) Profit from continuing operations 350 220 Profit from discontinued operations 4&5 30 200 Profit for the period 380 420 Other comprehensive income 0 0 Total comprehensive income 380 420 Example Ltd Notes to the financial statements For the year ended 31 December 20X3 (extracts) 20X3 20X2 4. Discontinued operation: analysis of profit C’000 C’000 The profit from discontinued operations is analysed as follows: • Revenue 150 790 • Expenses (100) (500) • Profit before tax 50 290 • Tax (60) (97) • Gains/ (losses) after tax 40 7 • Gain/ (loss on re-measurement to fair value less costs to sell 30 10 • Gain/ (loss) on disposal of assets in the/ the discontinued 20 0 operations • Tax on gains/ (losses) (10) (3) • Profit for the period 30 200 8.2 In the statement of cash flows In respect of discontinued operations, an entity shall disclose the following either on the face of the statement of cash flows or in the notes thereto ‘for all periods presented’ [para 33(c)]: • net cash flows from operating activities; • net cash flows from investing activities; and • net cash flows from financing activities. Example Ltd Notes to the statement of cash flows For the year ended 31 December 20X3 (extracts) 20X3 20X2 4. Discontinued operation C’000 C’000 Included in the statement of cash flows are the following net cash flows resulting from a discontinued operation: Net cash flows from operating activities (assumed figures) 5 6 Net cash flows from investing activities (assumed figures) 0 1 Net cash flows from financing activities (assumed figures) (8) (4) Net cash outflows (assumed figures) (3) 3 327 Chapter 9
  • 20. Gripping IFRS Non-current assets held for sale and discontinued operations 8.3 Other note disclosure 8.3.1 Components no longer held for sale (IFRS 5.37) Where the component is no longer ‘held for sale’, the amounts previously disclosed as ‘discontinued operations’ in the prior periods must be reclassified and included in ‘continuing operations’. This will facilitate better comparability. See the examples of disclosure provided in 8.1 and assume that the discontinued operation was first classified as such in 20X2, but that during 20X3 the criteria for classification as ‘discontinued’ were no longer met. Notice that the 20X2 figures shown below, whereas previously split into ‘continuing’, ‘discontinuing’ and ‘total’ (in 8.1) are now restated in one column. Although IFRS 5 does not require it, it is suggested that a note be included explaining to the user that a previously classified ‘discontinued operation’ has been reabsorbed into the figures representing the ‘continuing operations’ of the entity, thus explaining the re-presentation of the 20X2 figures. Example Ltd Statement of comprehensive income For the year ended 31 December 20X3 (extracts) 20X3 20X2 C’000 C’000 Restated Revenue 1 000 1 600 Expenses (400) (900) Profit before tax 600 700 Tax expense (220) (280) Profit for the period 380 420 Other comprehensive income 0 0 Total comprehensive income 380 420 The above amounts are assumed amounts: notice how they tie up with the previous explanatory examples in Option A and Option B. 8.3.2 Criteria met after the end of the reporting period (IAS 5.12) If the criteria for separate classification and measurement as ‘held for sale’ are met during the post-reporting date period, no adjustments should be made to the amounts and no reclassification of the assets as ‘held for sale’ should take place. This is treated as a non- adjusting event with the following disclosure being necessary: • a description of the non-current asset (or disposal group); • a description of the facts and circumstances leading to the expected disposal; • the expected manner and timing of the disposal; and • the segment (if applicable) in which the non-current asset (or disposal group) is presented. The note disclosure of an event after the reporting period might look like this: Example Ltd Notes to the financial statements For the year ended 31 December 20X3 (extracts) 4. Events after the reporting period On 15 February 20X4, the board of directors decided to dispose of the shoe division following severe losses incurred by it during the past 2 years. The division is expected to continue operations until 30 April 20X4, after which its assets will be sold on a piecemeal basis. The entire disposal of the division is expected to be completed by 31 August 20X4. 328 Chapter 9
  • 21. Gripping IFRS Non-current assets held for sale and discontinued operations 9. Summary IFRS 5 Non-current assets Discontinued held for sale operations Non-current assets held for sale Identification General If asset not expected Assets acquired with to be sold within 1 yr intention to sell Normal 6 criteria 3 scenarios and 2 criteria related criteria Measurement Cost model Revaluation model Assets acquired with intention to sell Initially: Initially: Initially: at cost at cost Lower of CA (cost) and FV – costs to sell Subsequently: Subsequently: Before reclassification: Before reclassification: Depreciate and impair Depreciate; revalue and impair When reclassifying: When reclassifying: Remeasure on Remeasure on revaluation cost model model Adjust to lower of CA or Adjust to lower of CA or FV FV – CtS – CtS Stop depreciating Stop depreciating Transfer to NCAHforS Transfer to NCAHforS Remeasure to latest FV – Remeasure to latest FV – CtS CtS (reversals of IL limited (reversals of IL limited to to accumulated IL’s) accumulated IL’s) 329 Chapter 9
  • 22. Gripping IFRS Non-current assets held for sale and discontinued operations Non-current assets held for sale No longer held for sale Transfer back to PPE Remeasure to lower of: • CA (had asset never been classified as NCAHforS); and • RA Resume depreciation Discontinued operations Identification Measurement Disclosure A component that has Same as for non- Statement of been disposed of or is current assets held comprehensive income: classified as held for for sale Face: sale and is: Total profit or loss from • Separate major line discontinued operations or area Notes or on the face:; • Part of a single Analysis of total profit disposal plan or Gain or loss on re- • Is a subsidiary measurement acquired to sell Gain or loss on disposal of assets Tax effects of above Changes in estimates Statement of cash flows: (face or notes) Operating activities Investing activities Financing activities Other notes: Components no longer held for sale Criteria met after the end of the reporting period 330 Chapter 9